Sei sulla pagina 1di 56

1

Kinetics of Rigid Bodies


Chapter

Till so far we have discussed Kinematics and Kinetics of particles i.e.


translation motion and Statics of particle and rigid body. In translation motion
all the particles forming a rigid body move in similar fashion and do not move
relative to each other. It enables us to represent the body as a particle. In
kinematics any one particle of the body can is used to represents translation
motion of the entire body but in kinetics the concept of mass center is used for
the purpose. All the mass of the body could be assumed concentrated on its
mass center and all the forces acting on a body could be assumed to act on the
mass center irrespective of their points of application. In the previous chapter
− Kinematics of Rotation motion we learnt that particles forming a rigid body
do not move in similar fashion and, while dealing with equilibrium of rigid
bodies we learnt that point of application of a force decides its rotational effect
known as torque; therefore, while dealing rotation motion shape and size of
the body has to be considered. A rigid body is system of large number of
particles, which keep their mutual distances unchanged in all circumstances.
Concepts introduced in dealing with system of particles will now be extended
with the restriction of constant mutual distances between particles to analyze
rotation motion i.e. mechanics of rigid bodies. In this chapter concept of
moment of inertial as inertia to rotation motion, torque of a force about an
axis, rotational equivalent of Newton’s laws of motion and energy and
momentum methods are introduced.

1. Concept of Rotational Every particle of a rigid body in rotation moves on circular paths about the
Inertia. axis of rotation; therefore, kinetics of an isolated particle moving on circular
path can provide some ideas regarding kinetics of rotation motion. Consider a
particle of mass m constrained by a string to move on a circular path of radius
G
r about a fixed point O in free space. A force F acting along the tangent to the
circular path imparts the particle an angular acceleration α in anticlockwise
G
maτ
G
G sense and tension force T provides necessary centripetal acceleration as
F
shown in Fig. 8.1. Equations of motion describing the motion of the article in
accordance with Newton’s laws are
G
T P mω 2r G G G
O
∑ Fτ = maτ → F = maτ
G G G
Fig. 8.1
∑F n = man → T = −mω 2 rP / O

Taking moments about the center O of all the forces acting on the particle
and then summing up them, the above equations yield
G G G G G G G G
( )
rP / O × F + rP / O × T = rP / O × maτ + rP / O × −mω 2 rP / O
2

The left hand side equals to resultant torque about the center O of all the
external force acting on the particle and the right hand side equals to moments
G G
of all the effective forces. The vector product rP / O × aτ in the first term of the
G
right hand side become r 2α and the second term on both the side vanishes.
The above equation reduces to
G G
∑τ o = mr 2α [8.1]

The above equation resembles Newton’s second law of motion and can be
considered as an equivalent in rotation motion. Its comparison with the
second law eq. 5.2 suggests following facts. Torque, angular acceleration and
the term mr2 play similar roles in rotation motion as the net force, the
acceleration and mass plays in translation motion. Total mass of a body in
translation motion is the measure of its inertia to translation motion, therefore
the sum of all terms mr2 arising due to all particle of the rigid body provides
suitable measure of its inertia to rotation motion. The inertial to rotation
motion is known as rotational inertia or more commonly moment of inertia.
Moment of inertia is rotational analogue of mass. It plays much the same
role in kinetics of rotation motion as mass does in of kinetics of translation
motion, determining the relationship between angular momentum and
angular velocity, torque and angular acceleration, and several other
quantities. Being a sum of several scalar terms of the form mr2, it is scalar
quantity. Its treatment as a scalar quantity is sufficient for analyzing plane
motion, but a more advanced tensor treatment is required to analyze
complicated motions like rotation about axis in rotation.

1.1.Moment of inertia of a We have learnt in the previous section that the moment of inertia of particle m
system of particles. moving of circle of radius r is expressed by a term mr2 and moment of inertia
of a system of several particles of masses m1, m2, ……. mi, …….. mn moving on
circular paths of radii r1, r2, ……. ri, …….. rn respectively about the same axis
O m1
is given by sum of moments of inertia of each individual particle. It is denoted
r1 by Io, where the subscript designates the axis of rotation.
Consider a system of particles all moving about an axis OO’ as shown in
mi
ri
r2
m2 the Fig. 8.2, moment of inertia of the system about the axis is given by
n
Io = ∑ (m r )
i =1
i i
2
[8.2]
mn
rn
O’
Like inertia in translation motion moment of inertia is also independent of
state of rotation motion. You can see that irrespective of direction and
Fig. 8.2 magnitude of their angular velocities, moment of inertia of each particle
contributes as a positive term in the final expression of eq. [8.2].

1.2.Moment of inertia of a A rigid body is continuous distribution of mass and can be assumed consisting
Rigid body. of infinitely large number of point particles. If one of the point particle of
infinitely small mass dm is at a distance r from the axis of rotation OO’, the
O
moment of inertia of this point particle is given by
dI o = r 2 dm
r
The moment of inertia of the whole body about the axis OO’ can now be
dm
obtained by integrating the above equation over the limits to cover whole of
the body.

I o = ∫ dI o = ∫ r 2 dm [8.3]
O’
Fig. 8.3 The portion of the body here the point particle of mass dm is known as
differential element. The differential element is not always selected as point
3

particle; generally it is selected to fulfill two criterions. First, its moment of


inertia dIo about the given axis must be known and second, by adding
infinitely large number of similar portions the whole body can be formed.
Familiarity with the calculus of one variable only at present level requires
selecting the differential element in such a way that the eq. [8.3] involves
integration with respect to one variable only. It is achieved by selecting the
differential element as an infinitely small length on a line mass distribution,
an infinitely thin line on a surface mass distribution and an infinitely thin
surface in a volume mass distribution same as we have done in calculation of
location of mass center.

1.2.Moment of Inertia of Moment of inertia of a rigid body as expressed by eq. [8.3] depends on the mass
Uniform Bodies. of the body and distribution of the mass relative to the axis of rotation. In
general, expression for moment of inertia contains product of two terms. One
of them is the mass of the body and the other is a characteristic dimension,
which depends on the manner how mass of the body is distributed relative to
the axis of rotation. Obviously for uniform bodies expression of moment of
inertia depends on their shape and location and orientation of the axis of
rotation. Based on these facts we can conclude
Ö If mass distribution is similar for two bodies about an axis, expressions of
their moment of inertia must be of the same form about that axis.
Ö If the whole body or any of its portions is shifted parallel to the axis of
rotation, moment of inertia remains unchanged.
In this section we find expressions of moment of inertia of some uniform
bodies of simple geometric shapes that are most common to practical use.
These bodies include a thin rod bent into an arc segment, a ring, a thin rod, a
sector of a disk, a disk, and a sphere each of uniform density. All the
expressions for moment of inertia, we deduce are relative to an axis passing
through the mass center of the body. Such an axis is also known as centroidal
axis. Latter on we learn two theorems − theorem of perpendicular axes and
theorem of parallel axes. These theorems enable us to find moment of inertia
about some other axis if moment of inertia is known about an axis. We denote
moment inertias of a body about its centroidal by IC.

Moment of inertia of a uniform thin rod bent into shape of an arc. Consider
a thin rod of mass m bent into the shape of an arc or radius r subtending angle
φ =0 θ at its center C as shown in Fig. 8.4. It is desired to find expression of its
rdφ moment of inertia about an axis passing through the center and perpendicular
φ to the plane of the arc. The arc is a line mass distribution; therefore the

C differential element should be an infinitely small length of it. It is shown by
r dark shaded portion of angular span dφ.
φ =θ Since the rod is uniform, mass dm of the differential element is

Fig. 8.4 Mass of the arc m


dm = × Angular span of the element = dφ
Angular span of the arc θ
Moment of inertia of the differential element about the axis
mr 2
dIC = r 2 dm = dφ
θ
Moment of inertia of the whole arc rod is now given by eq. [8.3]
mr 2 θ
I C = ∫ dI C → IC = ∫ dφ = mr 2 [8.4]
θ 0
4

Moment of inertia of a Ring. Moment of inertia of a ring about an axis passing


through the center and perpendicular to the plane of the ring (centroidal axis)
can be calculated in similar way as we have used for the arc. The ring is a line
mass distribution; therefore the differential element is an infinitesimally small
length of angular span dφ on the ring as shown by dark shaded portion in Fig.
rdφ 8.5.

C
φ φ =0 Since the ring is uniform, mass dm of the differential element is
r
Mass of the ring m
dm = × Angular span of the element = dφ
Angular span of the ring 2π

Fig. 8.4 Moment of inertia of the differential element about the axis
mr 2
dI C = r 2 dm = dφ

Moment of inertia of the whole ring is now given by eq. [8.3]
mr 2 2π
IC = ∫ dIC → IC = ∫ dφ = mr 2 [8.5]
2π 0

You can see that a completer ring and an arc (a portion of a ring) have similar
mass distribution relative to their centroidal axes; therefore, they both have
the same expressions for their moment of inertias.

Moment of inertia of a uniform thin rod. Consider a thin rod of length A and
mass m and its centroidal axis passing through the mass center C as shown in
Fig. 8.6. The rod is a line mass distribution; therefore the differential element
should be an infinitely small length of it. It is shown by dark shaded portion of
r = − 12 A dr r = 12 A
r length dr.
B C A
Since the rod is uniform, mass dm of the differential element is

Fig. 8.6 Mass of the rod m


dm = × Length of the element = dr
Length of the rod A

Moment of inertia of the differential element about the axis


m 2
dI C = r 2 dm = r dr
A
Moment of inertia of the whole rod is now given by eq. [8.3]
m 12 A 2 mA2
I C = ∫ dI C → IC = ∫ r dr = [8.6]
A −2A
1
12

Moment of inertia of a sector of a uniform disk. A disk is a thin circular


plate or lamina. A sector of a disk is a portion shown by shaded area in Fig.
8.7. It is desired to obtain expression of its moment of inertia about an axis
passing through its center C and perpendicular to the plane of the disk. The
dx sector is a surface mass distribution; therefore, the differential element should
be an infinitely thin line, whose moment of inertia about the axis is known,
x therefore it is assumed as an arc of radius x and width dx as shown by dark
shaded portion.
C θ
Since the sector is uniform, mass dm of the differential element is
r
Mass of the sector m 2m
dm = × Area of the element = 1 2 × ( xθ dx ) = 2 xdx
Fig. 8.7
Area of the sector 2
r θ r

From eq. [8.4], moment of inertia of the arc is


5

2m 3
dI C = x 2 dm = x dx
r2
Moment of inertia of the whole sector is now given by eq. [8.3]
2m r 3 mr 2
IC = ∫ dIC →
r 2 ∫0
IC = x dx = [8.7]
2

Moment of inertia of a uniform disk. Consider a disk of mass m and radius r as


shown in Fig. 8.8. It is desired to obtain expression of its moment of inertia
about an axis passing through the center and perpendicular its plane. It can be
calculated in the similar way as adopted for its sector. The disk is a surface
mass distribution, so the differential element should be line mass of known
moment of inertia about the axis. It is take as ring of radius x and width dx as
shown in the figure by a dark shaded portion.
Since the disk is uniform, mass dm of the differential element is
dx
Mass of the disk m 2m
x dm = × Area of the element = 2 × ( 2π xdx ) = 2 xdx
C r Area of the disk πr r
From eq. [8.5], moment of inertia of the ring is
2m 3
dIC = x 2 dm = x dx
r2
Fig. 8.8
Moment of inertia of the whole sector is now given by eq. [8.3]
2m r 3 mr 2
I C = ∫ dI C → IC = 2 ∫0
x dx = [8.8]
r 2
You can see that a complete disk and a sector of the disk both have similar
mass distribution relative to their centroidal axes; therefore, they both have
the same expressions for their moment of inertias.

Moment of inertia of a homogeneous cylinder. It is a uniform distribution


of mass in shape of a cylinder. It is desired to obtained expression of moment
dm of inertia of a cylinder of radius r and mass about its centroidal axis*, which
passes through its mass centre along the length of the cylinder. If the cylinder
IC is assumed made of several identical disks each of mass dm and radius r, all of
these disks have same moment of inertia dIC given by eq. [8.8].

dI C = 12 r 2 dm .

From eq. [8.3], the moment of inertia of the whole cylinder is


Fig. 8.9
IC = ∫ dIC → I C = 12 r 2 ∫ dm = 12 Mr 2 [8.9]

You can see that a cylinder and a disk have similar mass distribution relative
to their centroidal axes; therefore, they both have the same expressions for
their moment of inertias.

Moment of inertia of a homogeneous sphere. A homogeneous sphere or


simply a sphere is a uniform spherical distribution of mass. Consider a sphere
of mass m and radius r as shown in Fig. 8.10. It is desired to obtain expression
of moment of inertia of the sphere about an axis that coincides with any of its
diameter. In the figure C is the center of the sphere and the dashed line
passing through the center is the axis. The sphere is a volume mass
distribution; therefore the differential element should be a thin laminar

* A centroidal axis passes through centroids of each identical section of the body.
6

portion, whose moment of inertia about the given axis must be known. It is
taken in form of a disk of radius r, thickness dx as shown in the figure. The
pane of the disk is perpendicular to the axis and its center O is at a distance x
away from the center of the sphere.
C Since the sphere is uniform, mass dm of the differential element is
x
R
O r P Mass of the sphere 3m 2
dx dm = × Volume of the element = r dx
Volume of the Sphere 4 R3
From eq. [8.8], the moment of inertia of the disk is
Fig. 8.10 r 2 dm 3m 4
dI C = = r dx
2 8R3
Substituting r 2 = R 2 − x 2 in the above equation to reduce it into a function of
one variable, we have
r 2 dm 3m
( )
2
dI C = = R2 − x 2 dx
2 8 R3
By integrating the above function within the limits x = 0 to x = R to cover
lower half of the sphere, we get moment of inertia of the lower half. Having
similar mass distributions the lower and upper halves of the sphere have same
moment of inertias, therefore the moment of inertia of the whole sphere is
twice of that of the lower or upper halves.
3m
∫ (R )
x =R R 2
IC = 2∫ dI C → IC = 2 2
− x2 dx = 52 mR2 [8.10]
x =0 8 R3 0

Example 8.1. Find moment of inertia of a uniform rectangular plate of mass m and
dimensions A×b about an axis passing through its mass center and parallel to one of its
edges.
Solution. The plate and the axis parallel to its width b is shown in the adjoining
A figure. The axis coincides with the x-axis. The plate can be assumes consisting of
several identical rods each of mass dm and parallel to the length A of the plate. From
y
b eq. [8.6], moment of inertia of each of these rods is
Iy
A2dm
dI x =
Ix 12
x
From eq. [8.3], the moment of inertia of the whole plate about the x-axis is

A2 mA2
I x = ∫ dI x → Ix = ∫ dm = Ans.
12 12

mb2
Similarly moment of inertia about the y-axis is Iy = Ans.
12

O’ Example 8.2. Find moment of inertia of a uniform rod of mass m and length A about
θ an axis OO’ passing through its mass center and inclined with the rod at an angle θ as
shown in Fig. Ex. 8.2.
Io
O Solution. The rod is a uniform line mass distribution, therefore the differential
Fig. Ex. 8.2 element should be a small portion of length dx resembling a point particle of mass dm
at a distance x from the center of the rod. The distance of the differential element from
O’
the axis of rotation is r = x sin θ as shown in the adjoining figure.
r = x sin θ
θ m
The rod is uniform so mass dm of the differential element is dm = dx
x A
Io dx
O
7

m sin θ 2
Moment of inertia dIo of the differential element is dI o = r 2dm = x dx
A
The moment of inertia of the whole rod is

m sin θ 1A
mA2 sin 2 θ
I C = ∫ dI C → IC = ∫ x 2dx = Ans.
2

A − 12 A 12

O’ Alternate method. If a portion of a body is shifted anywhere parallel to an axis of


A θ P B rotation, moment of inertia of the body about that axis remains unchanged. If every
θ infinitely small portion of mass dm at a place P is shifted to P’ a distance OP cos θ , a
new uniform rod of the same mass but length Asinθ is formed. The new hypothetical
rod AB’ is perpendicular to the axes OO’ and is shown by dashed lines in the adjoining
Io figure. Moment of inertia of this rod and the original rod about the axis OO’ must be
O P’
B’ equal, so we have form eq. [8.6]

m ( AB ' )
2
mA2 sin 2 θ
Io = = Ans.
12 12

1.3.Theorems on Moment Moment of inertias of a rigid body about different axes may be different. Eq.
of Inertia. [8.3] can be used to find moment of inertia of a body about any axes, but it
requires lengthy calculations each time. There are two theorems known as
theorem of perpendicular axes and theorem of parallel axes, which greatly
simplify calculation of moment of inertia about an axis if moment of inertia of
a body about another suitable axis is known.

Theorem of Perpendicular Axes. This theorem is applicable for a rigid body


that lies entirely within a plane i.e. a laminar body or a rod bent into shape of
a plane curve. Consider a laminar body that lies in the x-y plane as shown in
z Fig. 8.11. An infinitesimally small portion of mass dm of the body lies at point
(x, y). The moment of inertia Ix, Iy and Iz of the body about the x, y and z-axis
can be expressed by the following equations.
O
r y x I x = ∫ y2 dm ; I y = ∫ x 2dm and I z = ∫ r 2 dm
x dm
Since r 2 = x 2 + y2 , the moment of inertia of the body about the z-axis can be
y
expressed by the following eq.
Fig. 8.11
Iz = I y + Ix [8.11]

For a planar body, the moment of inertia about an axis perpendicular to the
plane of the body is the sum of the moment of inertias about two perpendicular
axes in the plane of the object provided that all the three axes are concurrent.

Example 8.3. Find moment of inertia of a uniform rectangular plate of mass m and
dimensions A×b about an axis passing through its mass center and perpendicular to its
Iz z plane.
A
Solution. Let the plate is in the x-y plane with its mass center at the origin as shown
in the adjoining figure. It is desired to find moment of inertia about the z-axis. Its
y
b moment of inertias about the x-axis and y-axis are I x = 121 mA 2 and I y = 121 mb2 .
Iy
Substituting these values in the theorem of perpendicular axes, we have

x
Ix Iz = I y + Ix → Iz = 1
12 (
m A2 + b2 ) Ans.
8

Example 8.4. Find moment of inertia of a uniform disk of mass m and radius r about
one of its diameter.

z
Solution. In the adjoining figure a disk is shown with two of its diameter
Iz perpendicular to each other. These diameters are along the x and the y-axis of a
coordinate system. The x-axis is perpendicular to the plane of the disk and passes
Iy through its center is also shown.
C
x
Since the disk is symmetric about both the diameters, moment of inertias about both
the diameters must be equal. Thus I x = I y . Substituting this in the theorem of
Ix perpendicular axes, we have
x
Iz = I y + Ix → I z = 2I x = 2I y

From eq. [8.8], moment of inertia of the disk about the z-axis is I z = 12 mr 2 . Substituting
it in the above equation, we have

I x = I y = 12 I z = 14 mr 2 Ans.

Example 8.5. Find moment of inertia of a uniform ring of mass m and radius r about
one of its diameter.
Solution. Following method same as in the previous example, the moment of inertia
of a ring about one of its diameter is 12 mr 2 . Ans.

Theorem of Parallel Axes. This theorem also known as Steiner's theorem


can be used to determine the moment of inertia of a rigid body about any axis,
if the moment of inertia of the body about a parallel axis passing through mass
center of the body and perpendicular distance between both the axes is known.
z z' Consider a body of arbitrary shape and mass m shown in Fig. 8.12
Io IC
placed in a coordinate system whose origin is at O. Another
P coordinate system with its origin at the mass center C of the body is
O C y also shown. It is the center of mass frame. In both frames y-axes
y' coincide and x and z-axis are parllel. The coorindates of the mass
center in the former frame are (xC, 0, 0). Let the moment of inertia of
x x'
xC the body about the z-axis of the center of mass frame is known and
equals to IC. It is desired to obtain momentof inertia abou the z-axis of
Fig. 8.12
the other frame.
Let a particel P of mass dm forming the body is at point (x, y, z) with
respect to the origin O and (xP/C, yP/C, zP/C) with respect to the mass center C.
Transformation relations for coordinates in these frames are given by the
following equations.
G G G
x = xC + x P / C ; y = yP / C ; and z = zP / C [8.12]

The moment of inertia of the particle P about the z-axis of the frame with
origin at O is
dI o = x 2 dm + y2 dm [8.13]

The moment of inertia of the particle P about the z-axis of the center of mass
frame is
dI C = x P2 / C dm + yP2 / C dm [8.14]

Integrating the above function within limits to cover whole body and
rearranging the terms, we have

I C = ∫ dI C → ∫ (x )
2
IC = P /C + yP2 / C dm [8.15]
9

Substituting values from eq. [8.12] into eq. [8.13], we have


G G
dI o = xC2 dm + x P2 / C dm + 2xC ⋅ x P / C dm + yP2 / C dm

Integrating the above function within limits to cover whole body and
rearranging the terms, we have
G G
I o = ∫ dI o → ( )
I o = ∫ x P2 / C + yP2 / C dm + xC2 ∫ dm + 2xC ⋅ ∫ x P / C dm

The integral in the last term is sum of mass moments in center of mass frame,
which always equals to zero. By substituting value of IC form eq. [8.15], we
have
IO = I C + MxC2 [8.16]

The above equation is known as the theorem of parallel axes or Steiner's


theorem.
Ö Among all the parallel axes the moment of inertia of a rigid body about the
axis through the mass center is the minimum moment of inertia.
Ö The moment of inertia about any axis parallel to an axis through the mass
center is given by sum of moment of inertia about the axis through the mass
center and product term of mass of the body and square of the distance
between the axes.

The second term added to the moment of inertia IC about the centroidal
axis in eq. [8.16] can be recognized as the moment of inertia of a particle of
mass equal to that of the body and located at its mass center. It again reveals
that the plane motion of a rigid body is superposition of pure rotation about the
mass center or centroidal rotation and translation of its mass center.

Example 8.6. Find moment of inertia of a uniform ring, uniform disk, uniform
cylinder and uniform sphere each of mass m and radius r about their instantaneous
axis of rotation in rolling.
Solution. In rolling the instantaneous axis of rotation passes through the point of
contact P with the surface on which the body rolls. Each of these bodies has round
C vC section of radius r and can be represented in the adjoining figure. Denoting the
ω moment of inertia about the instantaneous axis of rotation by IP and through parallel
centroidal axis by IC, we have from the theorem of parallel axes.
P
I P = I C + Mr 2

By using expression for IC for the ring, the disk, the cylinder and the sphere form eq.
[8.5], [8.8], [8.9] and [8.10], we have

Ring I P = I C + Mr 2 = Mr 2 + Mr 2 = 2 Mr 2 Ans.

Disk I P = I C + Mr 2 = 12 Mr 2 + Mr 2 = 23 Mr 2 Ans.

Cylinder I P = I C + Mr 2 = 12 Mr 2 + Mr 2 = 23 Mr 2 Ans.

Sphere I P = I C + Mr 2 = 52 Mr 2 + Mr 2 = 75 Mr 2 Ans.

Example 8.7. Find expression for moment of inertia of a uniform cylinder of mass m,
radius r and length A about an axis passing through the mass center C and
perpendicular to the length of the cylinder.
Solution. Consider the cylinder shown in the adjoining figure. It is a volume mass
distribution so the differential element should be a surface mass. An infinitely thin disk
of mass dm and thickness dx shown in the figure is assumed as the differential
element.
10

Io dI = 1
r 2dm m
4
The cylinder is uniform, therefore mass dm of the disk is dm = dx
x dx A

dm Let moment of inertia of the disk about an axis along its vertical diameter is dI. It can
be written by using theorem of perpendicular axes and above expression for dm.
O
mr 2
dI = 14 r 2dm = dx
4A
x = − 12 A x = 12 A
Moment of inertia dIo of the disk about the axis passing through the mass center of the
cylinder and perpendicular to the length of the cylinder can be written by the theorem
of parallel axes and the above expression of the dI.

mr 2 m
dI o = dI + x 2dm → dI o = dx + x 2dx
4A A

On integrating the above within limits x = − 12 A to x = 12 A to cover the whole cylinder, we


obtain expression for the moment inertia about the desired axis.
1A
mr 2 1A
m 12 A 2 mr 2 mA 2
Io = ∫ dI o → Io = ∫ dx + ∫ x dx = + Ans.
2 2

− 12 A 4A − 12 A A −2A
1
4 12

Example 8.8. Find expression for moment of inertia of a uniform disk of mass m,
radius r about one of its secant making an angle θ with one of its diameter.
E
Solution. A disk, the secant OB and diameter OA are shown in the adjoining figure.
The secant OB is parallel to another diameter DE. Moment of inertia of the disk about
A C
θ one of its diameter is 14 mr 2 and hence moment of inertia I1 about the diameter DE.
O
Distance between the secant OB and the parallel diameter DE is rsinθ.
D
B
r sin θ
Substituting above information in the theorem of parallel axes, we have
I1 = 14 mr 2
I2 I 2 = I1 + m ( r sin θ ) →
2
I 2 = mr 2 ( 1
4
+ sin 2 θ ) Ans.

1.4.Moment of Inertia of According to eq. [8.2], moment of inertia about an axis is an additive quantity
Composite Bodies. and we can conclude the following facts.
Ö A composite body is made by joining two or more bodies. Moment of inertia
of a composite body about an axis is the sum of moment of inertias of its
component bodies about the same axis.
Ö If a body is made by removing a portion from a larger body, the moment of
inertia of the remaining body about an axis can be obtained by subtracting
moment of inertia about the same axis of the removed portion from
moment of inertia about the same axis of the original body.

Example 8.9. Find moment of inertia about centroidal axis of a bobbin, which is
Centroidal constructed by joining coaxially two identical disks each of mass m and radius 2r to a
axis cylinder of mass m and radius r as shown in the figure.
Solution. The bobbin is a composite body made by joining two identical disks
coaxially to cylinder. The moment of inertia I of the bobbin equals to the sum of
moment of inertias of the two disks and moment of inertia of the cylinder about their
centroidal axes. Using expressions for the moment of inertia for disk and cylinder from
Fig. Ex. 8.9
eq. [8.8] and [8.9], we have

I = 2 I disk + I cylinder → I =2 { m (2r ) } +


1
2
2 1
2
mr 2 = 92 mr 2 Ans.
11

Example 8.10. Find moment of inertia about one of diameter of a hollow sphere of
mass m, inner radius r and outer radius R.
Solution. The hollow sphere is assumed as if a concentric smaller sphere of radius r
is removed from a larger sphere of radius R. Thus the moment inertia of the hollow
sphere about any axes can be obtained by subtracting moment of the smaller sphere
from that of the larger sphere. As shown in the following figure.
Let the mass of the hollow sphere is m. Density of the material used is

y y 3m
I1 I2 ρ=
y
I2 (
4π R3 − r 3 )
Masses m1 and m2 of the smaller spheres are
O x O x O x
mR3 mr 3
m1 = ρ ( 4
3
π R3 ) =
R3 − r 3
; and m2 = ρ ( 4
3
π r3 ) =
R3 − r 3
I1 − I 2 = I
Using above values of the masses in eq. [8.10], moment of
inertias I2and I2 can be expressed. Subtracting I2 form I1 we have I.

I = I1 − I 2 → I = − =
5
2mR 2 2mr 2 2m R − r
5
( ) Ans.
5 5 5 R3 − r 3 ( )

Example 8.11. Find moment of inertia about one of diameter of a thin walled hollow
sphere of mass m and radius R.
Solution. A thin walled hollow sphere can be assumed as a thick walled hollow
sphere described in the previous example with limits r tending to R. Thus, we can
express moment of inertia I of the thin walled hollow sphere by the following equation.

I = lim
(
2m R5 − r 5 )
r→R
(
5 R −r 3 3
)
Using L' Hospital’s rule to calculate the limit, we have I = 23 mR2 Ans.

1.5.Radius of Gyration. The radial distance from a rotation axis at which the mass of an object could
be concentrated without altering the moment of inertia of the body about that
axis. If the mass m of the body were actually concentrated at a distance k from
the axis, the moment of inertia about that axis would be mk2 .

I
k= [8.17]
m

The radius of gyration has dimensions of length and is measured in


appropriate units of length such as meters.

2. Rotational Equivalent Eq. [8.1] resembling Newton’s second law presents a very restricted case of a
of Newton’s laws of particle moving on circular path with its center at the origin about which
Motion. torque was calculated. In this case the torque and angular acceleration vectors
are parallel. It is not always true. If torque is calculated about any point other
than center of the circle both the vector will no longer be parallel. But for rigid
bodies in plane motion we consider torque about the axis of rotation and find
an equation in which torque vector and the angular acceleration both point in
the same direction along the axis of rotation. In this section we introduce the
torque about the axis of rotation and explore the rotational equivalent of
Newton’s second law for a rigid body in plane motion.
12

2.1. Torque about a Point We have defined torque of a force about a point as the moment of the force
G G
and Torque about an about that point. If a force F acts at a point P of position vector rP the torque
Axes. G
τ o of the force about the origin O is given by the following equation.
G G G
τ o = rP × F [8.18]

z Consider a rigid body in which a rod AB passes through a straight


hole. The rod is held along the z-axis of a coordinate system so that the
α
G G
A body can rotate about the z-axis as shown in the Fig. 8.13. A force F is
G
τo G
applied at a point P. The torque τ o of the force about the origin O given
by the eq. [8.18] is shown in the figure. It is perpendicular to the plane
G
τz
G G
G
τ xy containing vectors F and rP . Certainly it is not parallel to angular
G G
C r acceleration vector. Its two components one τ z along the z-axis and the
G G G
F other τ xy in the x-y plane are shown in the figure. The component τ z
x P

B which is in the direction of the axis of rotation is parallel to the angular


G
rP acceleration vector and responsible for the rotation of the body about
G
O the z-axis. The other component τ xy will try to tilt the axis of rotation;
y
x but the forces holding the rod along the z-axis provide an additional
G
Fig. 8.13 torque to counterbalances it, maintaining the net torque equal to τ z
parallel to the angular acceleration vector in agreement with the eq. [8.1]. It is
G
equal to moment of the force F about the center C of the circular path which
the point P will follow. It is known as the torque of the force about the axis of
rotation.
When a body is in plane motion the net torque of all the forces including the
forces necessary to restrain rotation of the axis is along the axis of rotation. It is
known as torque about the axis. Torque of force about an axis of rotation equals
to the moment of force about the point where plane of motion of the point of
application of the force intersects the axis.
In analyzing plane motion we always consider torque about an axis under
consideration and in rest of the book by the term torque of force we mean
torque about an axis.

2.2. Torque of Distributed A distributed force does not act on a point but it acts on whole or a part of a
Force. body. When you press a wall with your palm the force applied by you is
actually the resultant of all the forces acting at every contact point, force of
friction is resultant of all the forces acting at every contact point between the
surfaces in contact, weight of a body is the resultant of all the gravitational
forces acting on all the particles of the body. Force applied by your palm on a
wall and the friction force between two surfaces are examples of a force
distributed on an area whereas the force of gravity is an example of a force
distributed into volume of a body.
If we can represent distributed force acting on a body by a single force
equal to a net resultant of the distributed force, analyses of its effect on the
motion of the body would be greatly simplified. As far as translation motion is
concerned all the forces whether distributed or not are assumed to act on the
mass center of the boy, but while analyzing rotation motion point of
application of the forces must be taken into account. There is no problem with
finding the net resultant of all the distributed forces, but it requires some
more investigations to find an appropriate point of application for net
resultant force in order to maintain its effect on rotation motion of the body
unaltered. Obviously the point of application of distributed force is a
mathematical point at which, if the net resultant of a distributed force is
assumed to act, its torque about a point or an axis must be equal to the torque
of the actual distributed force. The point of application of a distributed force is
also known as the force center.
13

Thus torque of a distributed force is either calculated by integrating torque


of the every minute force acting on an infinitely small part of the body under
consideration or by assuming a net resultant of the distributed force acting on
its force center.

Center of Gravity and In chapter-4, while dealing with equilibrium of rigid bodies, the concept of
Mass Center. torque, introduced for force acting on a point was extended for force of gravity
by assuming it acting at the mass center of the body. It is fairly correct for
bodies much smaller than the size of the earth so that gravitation field
everywhere within the body can be assumed fairly uniform. Or in other words
if gravitation field of the earth is uniform within the volume of a body, its mass
center and force center of gravity coincide. The force center of gravity is known
as center of gravity.
Imagine a particle of mass mi in a rigid body
G
y y of mass M. The position vector of the particle is ri
G
C CG and that of the mass center of the body is rC . The
net torque about the origin of weights of all the
G mi G
rC rCG Mg particles of the body must be equal to the torque
G
ri
of the net weight acting at the center of gravity of
mig the body as shown in Fig. XX.
O x O x n
G G G G
Gravity as distributed Force It is assumed to act on the ∑ (r × m g ) = r
i i CG × Mg [8.19]
center of gravity. i =1

Fig. 8.14
If gravity is uniform we can write
G G G G G G G G
∑ ( r × m g ) = ∑ (m r ) × g = Mr
i i i i C × g = rC × Mg ; therefore the above equation
reduces to
n
G G G G G G
rCG × Mg = rC × Mg = ∑ (r × m g )
i =1
i i [8.20]

Example 8.12. A uniform disk of mass M and radius R rotating about a vertical axis
passing through its center and perpendicular to its plane is placed gently on a rough
horizontal ground, were coefficient of friction is μ. Calculate torque of the frictional
forces.
Solution. When the disk rotates on the ground, kinetic friction acts at every contact
Fig. Ex. 8.12
point. Since the gravity acts uniformly everywhere and the disk is also uniform, the
normal reaction form the ground is uniformly distributed over the entire contact area.
Consider two diametrically opposite identical portions A and B of the disk each of mass
df dm at distance r from the center as shown in the adjacent figure. The normal reaction
C
A r r B form the ground on each of these portions equals to their weights and hence frictional
df forces are df = μ dmg . Friction forces on these two and on all other diametrically
opposite portions of the disk are equal and opposite, therefore net resultant friction
force on the disk is zero. But torque of friction force on every portion is in same
direction and all these torques add to contribute a net counterclockwise torque about
the axis.
Consider a ring of radius r and width dr shown by dashed lines. Net torque dτ C of
friction force on this ring can easily be expressed by the following equation.

⎛ Mass of the disk ⎞ ⎛ 2 Mrdr ⎞


dτ C = r μ ( mass of the ring ) g = r μ ⎜ × Area of the ring ⎟ g = r μ ⎜ ⎟g
⎝ Area of the disk ⎠ ⎝ R
2

Integrating both sides of the above equation, we have
2μ Mg R
τC =
R2 ∫
r =0
r 2 dr = 23 μ MgR Ans.
14

Consider a rigid body in plane motion i.e. in rotation about an axis in


2.1.Dynamics of Rigid translation. Let the axis of rotation always remain parallel to the z-axis of a
Body as System of
coordinate system representing an inertial reference frame. All particles of the
Particles.
body move in planes parallel to the x-y plane. Thus every section of the body
parallel to the x-y plane are in similar kind of motion and can be used to
G
G represent plain motion of the body. In the Fig. 8.15 (a) such a section of a body
aC
y ai / C in plain motion is shown. Consider a particle of mass mi of the body. The whole
mi
G
ai body can be assumed to be made of large number of similar particles. Let the
resultant of all the forces applied on the particle by agencies external to the
G G
aC body is Fi , the force applied by another particle mj of the body is represented
C G G
by fij and acceleration of the particle relative the inertial frame by ai .
Newton’s second law of motion suggests that translation motion of the particle
O x can be described by the following equation.
Fig. 8.15 (a)
G n G G
Fi + ∑
j =1; j ≠ i
fij = mi ai [8.21]

Here the second term expresses the resultant of forces applied by all other
particles of the body and represents the total internal force on the particle mi.
G G
Every internal fore fij on particle mi due to particle mj and f ji on the particle mj
due to particle mi constitute Newton’s third law pair of equal and opposite
force, therefore the sum all these internal forces on all the particles must be
zero. Keeping this fact in mind and denoting the mass of the whole body by M
G
and acceleration of the mass center C by aC relative to the inertial frame, the
Newton’s second law for translation motion the body is represented by the
following equation.
n G n
G G
∑ F = ∑ (m a ) = Ma
i =0
i
i =0
i i C [8.22]

G The plane motion of the body can be represented as superposition of


G aC
y ai / C translation of its mass center and simultaneous rotation about the mass
G centre. The translation of the mass center is described by the eq. [8.22]. The
mi ai
G rotation about the mass center is represented by circular motion of all of its
ri / C G G
G
aC
particles with angular acceleration α and angular velocity ω which are equal
G C
ri G to angular acceleration and angular velocity of rotation of the body. The
α G
G particle of mass mi is shown moving on circular path of radius ri / C . Here for
rC G
the sake of simplicity only the radial acceleration term of acceleration ai / C is
O x
Fig. 8.15 (b) assumed zero. We will see later in eq. [8.25], that the radial acceleration term
G
appears as cross product with ri / C which becomes zero.

Taking moments about the origin of terms of both the sides of eq. [8.21]
and substituting in eq. [8.22], we have
G
∑ (r × F ) = ∑ (r × m a )
n n
G G G
i i i i i [8.23]
i =0 i =0

It is not difficult to show that net moment of all the internal forces about
y
the origin or any other point must be zero. Consider two particle mi and mj
G G
mi G attracting each other by equal and opposite forces fij and f ji as shown in the
fij
G Fig. 8.16. The sum of their moments about the origin is zero. As explained in
G f ji
ri mj the following equation.
G G G G G G G G G G G
G
rj
G G
( )
ri × fij + rj × fji = ri × fij + rj × − fij = ( ri − rj ) × fij = ri / j × fij = 0
O x
Fig. 8.16 Since sum of moments of action-reaction pair of two internal forces is zero,
sum of moments all such pairs and hence all internal forces about any point
must be zero.
15

The left hand side of eq. [8.23] equals to net moment or torque of all the
n
G
external force about the origin. Denoting it by ∑τ
i =0
o and substituting relative
G G G G G G
motion equations ri = rC + ri / C and ai = aC + ai / C , we obtain
n n n
G G G G G G G
∑τ o = ∑ {mi ( ri × ai )} = ∑ {mi ( rC + ri / C ) × ( aC + ai / C )}
i −1 i −1 i −1

n n
G G G G G G G G G
∑τ o = ∑ {mi ( rC × aC + rC × ai / C + ri / C × aC + ri / C × ai / C )}
i −1 i −1

n n n n n
G G G G G G G G G
∑τ
i −1
o = rC × ∑ ( mi ) aC + rC × ∑ ( mi ai / C ) + ∑ ( mi ri / C ) × aC + ∑ ( mi ri / C × ai / C )
i −1 i −1 i −1 i −1

From eq. XX and YY, the second and the third terms of the right hand side
each become zero. So we have
n n
G G G G G
∑τ
i −1
o = rC × MaC + ∑ ( ri / C × mi ai / C )
i −1
[8.24]

The product of mass and acceleration vector of a particle being resultant of


all the forces acting on it is known as effective force. The above equation
explicates that the net torque about the origin of an inertial frame of all the
external forces acting on a body equals to moment of effective force due to
translation of mass center and sum of moments of effective forces of all the
particles due to their motion with respect to mass center.
Since distances between all the particle on a rigid body is fixed, the motion
relative to mass center can be nothing but the rotation of the body about an
axis passing through the mass center. It is known as centroidal rotation. The
eq. [8.24] also asserts that the plane motion of a rigid body is superposition of
translation of its mass center and pure rotation about the mass center or
centroidal rotation.
G G G G
Substituting ai / C = α × ri / C − ω 2 ri / C from the relative motion equation in eq. [8.24]
we have

{G }
n n
G G G G G G
∑τ o = rC × MaC + ∑ ri / C × mi (α × ri / C − ω 2ri / C )
i −1 i −1

{ )}
n n n
G G G G G G G G
∑τ o = rC × MaC + ∑ {ri / C × mi (α × ri / C )} − ∑ ri / C × mi ω 2 ri / C ( [8.25]
i −1 i −1 i −1

The last term involves cross product of a vector with itself, hence vanishes.
Now solving the second term as vector triple product† we have
n
G G G ⎧n ⎫G
∑τ o = rC × MaC + ⎨∑ mi ri2/C ⎬α ( ) [8.26]
i −1 ⎩ i −1 ⎭
Since moment of inertia IC of the body about the axis through the mass center
equals is given by
n
I C = ∑ mi ri2/C ( ) [8.27]
i −1

The eq. [8.26] reduces to


n
G G G G
∑τ
i −1
o = rC × MaC + I Cα [8.28]

G G G GG G GG G

( )
a × b × c = ( a.c ) b − a.b c ( )
16

If torque of all the external forces is taken about the mass center of the
body, the first term on the right hand side vanishes and the equation reduces
to
n
G G
∑τ
i −1
C = I Cα [8.29]

The above equation describes centroidal rotation i.e. rotation of the body
about an axis passing through the mass center.

Superposition of translation of mass center and simultaneous Centroidal


Rotation. Plane motion of rigid body can be analyzed as superposition of
translation of mass center and simultaneous centroidal rotation. Here the
centroidal rotation represents rotation of the body about an axis passing
G through the mass center and parallel to the actual axis of rotation. The eq.
F1
G [8.22] and [8.28] provide the way to write equation of motion of a body in plane
Fi
G motion in terms of translation of mass center and centroidal rotation. The
I Cα G G G
MaC term MaC is known as effective force and ICα effective torque about the mass
C center. The Fig. 8.17 represents the free body diagram of a body in plane
G
F2
G
Fn
motion and equivalent effective force and effective torque about the mass
center.
Fig. 8.17 n G G
Translation of mass center ∑F
i =0
i = MaC [8.22]

n
G G
Centroidal Rotation ∑τ
i −1
C = ICα [8.29]

Rotation about a fixed axis. A body is in rotation about an axis passing


y
G
ai
through point, which is fixed relative to the inertial frame, is said to be pivoted
mi
or hinged at the fixed axis. A body pivoted at an axis passing through point P,
G
ri / C which is at rest relative an inertial frame represented by a coordinate system
G
aC is shown in Fig. 8.18. Here for simplicity only tangential terms are shown.
G C
G
ri / P
rC / P Let under the action of several external forces the body starts rotation with
P G angular acceleration α in clockwise sense. Every particle mi of the body is now
α
in motion of circular path of radius ri / P with its center at P. Since point P is
stationary relative to the inertial frame, another frame with its origin at P is
O
x also an inertial frame; therefore from eq. [8.28], we have
Fig. 8.18 n
G G G G
∑τ
i −1
P = rC / P × MaC / P + I C α

G G G G
Substituting aC / P = α × rC / P − ω 2 rC / P in the above equation, we have
n
G G G G G G G
∑τ
i −1
P = MrC / P × (α × rC / P ) − M ω 2 ( rC / P × rC / P ) + ICα

The second term on right hand side involves cross product of a vector with
itself; therefore vanishes. Now by simplifying the first term as vector triple
product, we have
n
G G
∑τ P (
= MrC2/ P + I C α )
i −1

Applying the parallel axes theorem I P = MrC2 / P + I C , we have


n
G G
∑τ
i −1
P = I Pα [8.30]
17

Rotation about instantaneous axis of Rotation. Following analysis similar


to rotation about fixed axis, it can be shown that the eq. [8.30] is also
applicable if instead of a fixed axis the instantaneous axis of rotation passes
through the point P.

2.2. Force and Torque Plain motion of a rigid body includes fixed axis rotation and rotation about an
equations in Plane axis in translation. In the previous section we have learnt how to write force
Motion. and torque equations to analyze kinetics of plane motion. In this section
strategy applicable to force and torque equations i.e. the rotational equivalent
of the Newton’s second law is summarized.

Ö Rotation about fixed axis passing through the mass center. Rotation of
bodies pivoted such that the axis of rotation is fixed in space and passes
G through the mass center C comes in this category. In Fig. 8.19, a body
F1
G which can rotate about a fixed axis passing through its mass center C is
Fi G
I Cα
shown. The mass center of the body does not move hence the body does not
have any translation motion. Such kind of rotation motion is known as
C C pure rotation. Torque equation describing centroidal rotation should be
G G used to deal with.
F2 Fn
n
G G
Fig. 8.19 ∑τ
i −1
C = I Cα [8.29]

Ö Rotation about fixed axis not passing through the mass center. If a body
is rotating about an axis, which is fixed in space but does not pass through
G
R the mass center, the rotation of body comes in this category. In Fig. 8.20, a
G
G P maC body which can rotate about a fixed axis through point P is shown. It is
P Fi
G
I pα
easy to conceive that as the body rotates its mass center will move on a
G
circular path of radius rP / C . The mass center of the body is in translation
C C G
G
motion with acceleration aC . This kind of motion is known as rotation
G
F1 Fn about fixes axis but generally it is also called pure rotation.‡ To deal with
Fig. 8.20 this kind of situation eq. [8.22] and [8.30] should be used.
n G G G G G
Translation of mass center ∑F
i =0
i = MaC = M α × rC / P − M ω 2 rC / P [8.22]

n
G G
Rotation about the fixed axes ∑τ
i −1
P = I Pα [8.30]

G
Here left hand side of eq. [8.22] includes the reaction force R applied by the
pivot.

G Ö Rotation about axis in translation. Rotation of bodies about an axis in


F1
G
translation motion can be dealt with either as superposition of translation
Fi
G of mass center and centroidal rotation or assuming pure rotation about the
I Cα G
MaC instantaneous axis of rotation. It employs use of eq. [8.22] and [8.29] as
C shown in Fig. 8.17.
G G n G G
∑F
F2 Fn
Translation of mass center i = MaC [8.22]
i =0
Fig. 8.17
n
G G
Centroidal Rotation ∑τ
i −1
C = I Cα [8.29]

‡Though pure rotation and rotation about a fixed axis differ in the sense that in pure rotation
the mass center does not moves, yet this difference is ignored and we usually refer both the
terms interchangeably.
18

This kind of situation can also be dealt with considering it rotation about
IAR. It gives sometimes quick solutions, especially when IAR is known and
forces acting at the IAR are not asked.

Example 8.13. A block of mass m is suspended with the help of a light cord wrapped
over a cylindrical pulley of mass M and radius R as shown in the figure. The system is
released from rest. Find the angular acceleration of the pulley and the acceleration of
the block.
Solution. After the system is released, the block is in translation motion and the
Fig. Ex. 8.13 m pulley in rotation about an axis passing through its mass center i.e. in pure rotation.
Let the block moves vertically down with acceleration a pulling the cord down and
causing the pulley to rotate clockwise. Since the cord is inextensible every point on its
vertical portion and point of contact P of the pulley move down with acceleration a as
P
α shown in the adjacent figure. It is the tangential acceleration of point P so the angular
a = αR
acceleration α of the pulley rotating in clockwise sense is given by
a = αR (1)
The forces acting on the pulley and on the block are shown in their free-body diagrams
along with the effective torque I Cα of the pulley and effective force ma of the block.
Here T is the tension in the string, R is the reaction by the axel of the pulley, Mg is
weight of the pulley and mg is weight of the block.
R
The pulley is in rotation about fixed axis through its mass center so we use eq. [8.29].
a = αR G
G
P I Cα
∑τ C = ICα → TR = I Cα
T
After substituting I C = 12 MR2 and α from eq. (1), we have
Mg
T = 12 Ma (2)

T
The block is in translation motion, so we use Newton’s second law
G G
m
∑ F = ma → mg − T = ma (3)
mg ma
From equation (2) and (3), we have
2mg
Acceleration of the block a= Ans.
M + 2m
From eq. (1) and the above, we have
2mg
Angular acceleration of the pulley α= Ans.
R ( M + 2m )

Example 8.14. A rope of uniform mass m and length A is wrapped in single layer
around a cylindrical pulley of mass M and radius R, which can rotate freely. Find the
angular acceleration of the pulley as a function of length x of the hanging part of the
rope. Assume the rope thin enough.
x Solution. The hanging part of the rope is in translation motion but its portion of
length A − x wrapped on the pulley in shape of a ring of radius R rotates together with
Fig. Ex. 8.14 the pulley about a fixed axis passing through the common mass center of this portion
and the pulley. Therefore we can treat the pulley and the portion of rope wrapped on
the pulley as a composite body as shown in the figure. The moment of inertia IC of this
The pulley-rope
composite body composite body is

MR 2 m ( A − x ) 2 ⎪⎧ M m ( A − x ) ⎪⎫ 2
I C = I pulley + I ring of rope = + R =⎨ + ⎬R (1)
2 A ⎩⎪ 2 A ⎭⎪

Let the hanging portion of he rope is moving down with acceleration a. Since the rope is
inextensible every point on its vertical portion and point of contact P of the pulley move
α P down with acceleration a as shown in the adjacent figure. It is tangential acceleration
a = αR of point P; therefore the pulley-rope composite body rotates clockwise with angular
acceleration α, which is given by the following equation.
19

α = a/R (2)
The forces acting on the pulley-rope composite body and on the hanging portion of the
rope are shown in their free-body diagrams along with the effective torque I Cα on the
composite body and effective force mxa / A on the hanging portion of the rope. Here T is
the tension force at point of contact P, R is the reaction by the axel of the pulley, Mg is
R weight of the pulley and mg is weight of the hanging portion of the rope.
The pulley-rope composite body is in rotation about fixed axis through its mass center
I Cα
P so we use eq. [8.29].
G G
T
∑τ C = ICα → TR = I Cα
Mg
After substituting IC from eq. (1), we have

⎧⎪ M m ( A − x ) ⎫⎪
T =⎨ + ⎬α R (3)
⎪⎩ 2 A ⎪⎭

T The hanging rope is in translation motion, so we use Newton’s second law


G G mx mx
∑ F = ma → A
g −T =
A
a (4)
x mxg mxa
A A
From equation (2), (3) and (4), we have
2mgx
Angular acceleration of the pulley α= Ans.
AR( M + 2m)

Example 8.15. Two blocks are connected by a string that passes over a pulley of radius
m1 R and moment of inertia I. The block of mass m1 slides on a frictionless, horizontal
surface; the block of mass m2 is suspended from the string. Find the acceleration of the
blocks and the tensions in the horizontal and vertical portions of the string. Assume
that the string does not slip on the pulley.

m2 Solution. Both the block are in translation motion with the same acceleration a
Fig. Ex. 8.15
together with the string, which passes over the pulley without sliding causes it to rotate
with angular acceleration α about its fixed centroidal axis. Tangential acceleration of a
point on the rim of the pulley equals to the acceleration of the string, therefore
a = αR (1)
The pulley is not an ideal one. It has mass and moment of inertia. Since its weight and
T1 Iα reaction from the axel pass through the axel and do not create any torque about the
axel, the tensions in string on both sides of the pulley must be unequal in magnitude to
apply a net torque on the pulley. Tension forces are shown in the adjacent figure along
T2
with the effective torque Iα. As the pulley is in rotation motion only, its weight and
reaction from the axel are not shown. Applying torque equation about the center of the
pulley, we have
G G
∑τ C = ICα → (T2 − T1 ) R = I α
Substituting α from eq. (1), we have
Ia
T2 − T1 = (2)
R2
Forces acting on the blocks and the effective forces are shown it the adjacent figure.
m1 T1 m1a
Since weight and the normal reaction for m1 balance each other and do not contribute
to its acceleration, they are not shown.
Applying Newton’s laws of motion to block of mass m1, we have
T2
G G
m2 ∑ F = ma → T1 = m1a (3)
m2a Applying Newton’s laws of motion to block of mass m2, we have
m2g
G G
∑ F = ma → m2 g − T2 = m2a (4)
20

From eq. (2), (3) and (4), we have


m2
Acceleration a of the blocks a= Ans.
m1 + m2 + I R 2

m1m2 g
Tension in horizontal portion of string T1 = Ans.
m1 + m2 + I / R 2

(m1 + I / R 2 )m2 g
Tension in horizontal portion of string T2 = Ans.
m1 + m2 + I / R 2

Example 8.16. A cylinder of radius r and mass m rests on two horizontal parallel
corners of two platforms. Both the platforms are of the same height. Platform B is
suddenly removed. Assume friction between the corner of the platform A and cylinder
to be sufficient enough to prevent sliding. Determine angular acceleration of the
θ θ cylinder immediately after the removal of the platform B.
A B Solution. Since the cylinder does not slide at the point of contact with the corner of
platform A, it rotates about fixed axis through the point of contact in subsequent
Fig. Ex. 8.16 motion. Torque eq. [8.30] should be used.
Forces acting on the cylinder and the effective torque are shown in the adjacent
figure. Since forces acting at the point of contact does not contribute any torque about
it, the normal reaction form the corner and the friction force are not shown in the free
body diagram.
Applying the torque equation about the fixed axis through P, we have
G G
∑ τ P = I Pα → mgr sin θ = I Pα (1)

r θ
Applying the theorem of parallel axes and expression for moment of
P P I Pα
r sin θ
inertia about centroidal axes, we obtain moment of inertia IP about an axis
A A through the point P.

I P = I C + mr 2 → I P = 12 mr 2 + mr 2 = 23 mr 2 (2)

Substituting IP from eq. (2) in (1), we have

2 g sin θ
Angular acceleration of the cylinder α= Ans.
3r

Example 8.17. A thread is wrapped around a uniform disk of radius r and mass m.
One end of the thread is attached to a fixed support on the ceiling and the disk is held
stationary in vertical plane below the fixed support as shown in the figure. When the
disk is set free, it rolls down due to gravity. Find the acceleration of the center of the
disk and tension in the thread.
Solution. The point P, where the thread leaves the disk is always at instantaneous
Fig. Ex. 8.17 rest; therefore the disk can be assumed rolling without slipping with ICR at point P.
G
Acceleration of the mass center aC and angular acceleration of the disk are shown in
the adjacent figure. Applying condition for rolling on stationary surface, we have
α
G G G
P
G
C aC = α × rC / P → aC = α r (1)
aC
The disk rolls down on the vertical stationary thread. Its motion can either be analyzed
as superposition of translation of the mass center and simultaneous centroidal rotation
or a pure rotation about ICR. Since tension, which acts at the ICR is asked; we prefer
superposition of translation of the mass center and simultaneous centroidal rotation.
Forces acting on the disk are tension T applied by the thread at point P and weight of
T
ΙCα ΙCα the disk. These forces and the effective force maC and effective torque ICα are shown in
P
P C C the adjacent figure.
maC
Applying Newton’s second law for translation of mass center, we have
G G
mg
∑ Fi = MaC → mg − T = maC (2)

Applying torque equation for centroidal rotation, we have


21

G G
∑τ C = I Cα → Tr = I Cα
Substituting 1
2
mr 2 for IC and α form eq. (1), we have

T = 12 maC (3)

From eq. (2) and (3), we have


Acceleration of the mass center aC = 23 g Ans.

Tension in the string T = 13 mg Ans.

Example 8.18. Consider again the setup of the previous example. In stead of fixing the
thread at the ceiling, it is pulled vertically by a constant force F = 180 N as shown in
the figure. If the disk has mass 15.0 kg and radius 0.5 m, find angular acceleration of
the disk and acceleration of the thread.
Solution. The thread is moving up and the disk is rolling down the thread. The
situation is similar to rolling on a moving surface. Since the thread is inextensible its
Fig. Ex. 8.18 point P where it leaves the disk moves up with the same acceleration as all other points
on the vertical portion of the tread. Let the point P is moving up with acceleration aP,
G y mass center of the disk is moving down with acceleration aC and the disk is rotating
aP
α
x with angular acceleration α in the clockwise sense as shown in the adjacent figure.
P C
G
aC
Applying the relative motion equation, we have
G G G
aC = aP + aC / P → ( )
−aC jˆ = aP jˆ + −α kˆ × riˆ ⇒ aC = α r − aP (1)

The above relation could also be written by inspection. Since mass center C moves down
with acceleration αr relative to the point P, which is moving upwards with acceleration
aP; therefore, relative to the ground aC = α r − aP .

F = 180 N The disk is in rotation about an axis in translation. Its motion should be analyzed as
translation of the mass center and simultaneous centroidal rotation.

ΙCα Forces acing on the disk are its weight and tension equal to the applied force. These
P C
P
C
forces, the effective force maC and the effective torque ICα are shown in the adjacent
maC figure.

150 N
Applying Newton’s second law for translation of mass center, we have
G G
∑ Fi = MaC → 150 − 180 = 15.0aC ⇒ aC = −2.0 m/s2 (2)

The minus sign represents that the mass centre accelerates upwards. Since the upward
force F is greater than the downward force weight, the direction of aC could be
estimated without solving eq. (2).
Applying torque equation for centroidal rotation, we have
G G
∑τ C = ICα → Fr = I Cα
Substituting 1
2
mr 2 for IC , we have angular acceleration of the disk

2F 2 × 180
α= = = 48 rad/s2 Ans.
mr 15.0 × 0.50

Substituting above value of α and aC form eq. (2) in eq. (1), we have

Acceleration of the thread aP = α r − aC = 48 × 0.5 − ( −2 ) = 26 m/s2 Ans.

Example 8.19. The arrangement shown consists of two identical, uniform disks, each
of mass m and radius r, on which a light thread is wound. The lower disk B is initially
P A held keeping the system in rest. When it is released, it descends rolling down the
thread, which gets unwrapped from the disk A causing it to rotate clockwise. Find
angular acceleration and acceleration of mass center of the lower disk. The friction in
the axle of the upper disk is negligible
C B
Fig. Ex. 8.19
22

Solution. When the system is released, tension in the thread causes the disk A to
rotate clockwise about its fixed axel. Let its angular acceleration be αA. As a result
αA
P A point A has tangential acceleration a A = α A r downwards as shown in the adjacent
a A = α Ar figure. Since the tread is inextensible the point B on it also descends with the same
acceleration. So we have
αB
C B a A = aB = α A r (1)
aB = a A = α A r
The thread does not slip over any of the disks, therefore disk B rolls down on the
aC
descending thread rotating in anticlockwise sense. If we denote its angular acceleration
by αB, it is obvious to say that its mass center C descends with acceleration
aC / B = α B r relative to the point B. By applying relative motion equation, we have
G G G
aC = aB + aC / B → aC = aB + aC / B

Substituting aB = α r from eq. (1) and aC / B = α B r , we have

aC = (α A + α B ) r (2)

ΙPαA The disk A is in pure rotation about its axel P. The reaction at its axel and its weight do
P AP not apply any torque about the axel so they are not shown in its free body diagram. The
tension T creating torque about the axel and the effective torque I Pα A are shown in the
T
adjacent figure. Applying torque equation for pure rotation, we have
G G
∑ τ P = I Pα → Tr = I Pα A

Substituting 1
2
mr 2 for IP , we have

T = 12 mrα A (3)

The disk is in rotation about an axis in translation. Its motion can be analyzed as
T superposition of translation of mass center and simultaneous centroidal rotation.
ΙCαB
Forces acting on it, effective force maC and the effective torque I Cα B are shown in the
C AC
adjacent figure.
maC
Applying Newton’s second law for translation of mass center, we have
mg
G G
∑ Fi = MaC → mg − T = maC

Substituting aC form eq. (2), we have

mg − T = m (α A + α B ) r (4)

Applying torque equation for centroidal rotation, we have


G G
∑τ C = ICα → Tr = I Cα B

Substituting 1
2
mr 2 for IC , we have

T = 12 mrα B (5)

From eq. (3) and (5), we know that angular accelerations of both the disks are equal in
magnitudes.
α A = αB (6)

From eq. (3), (4) and (6), we have


2g
Angular accelerations of both the disks α A = αB = Ans.
5r
4mg
Acceleration of mass center of disk B aC = Ans.
5

Example 8.20. A uniform rod AB of mass m and length A is suspended in horizontal


position with the help of two strings as shown in the Fig. Ex. 8.20. The string
supporting the end B is cut. Find acceleration of the mass center and end A
A B
immediately after the string is cut.
Fig. Ex. 8.20
23

Solution. After the string is cut forces acting on the rod are tension in the string A
and weight of the rod. Both of these forces are in vertical direction so acceleration of the
mass center C must be vertically downwards. The string is inextensible so the point A
can have acceleration only in horizontal direction. Let acceleration of the mass center C
is denoted by aC downwards and acceleration aA of the point A towards the left as
A C B
shown in the adjacent figure.
aA α y
aC
Applying the relative motion equation, we have
x
G G G G G G JJJJG JJJJG
aC = a A + aC / A → aC = a A + α × AC − ω 2 AC

Immediately after the string is cut the rod cannot acquire any
angular velocity. So the last term in the above equation vanishes.
Now we have

( ) ( Aiˆ )
−aC jˆ = −a Aiˆ + −α kˆ × 1
2

Comparing x and y-components on both the sides we have


aC = 12 α A (1)

aA = 0 Ans.

The rod is in plane motion, which can be analyzed as superposition of


T ICα translation of the mass center and simultaneous centroidal rotation. The
C
forces acting on the rod immediately after the string at B is cut, the
A B
mg y maC effective force maC and the effective torque ICα are shown in the adjacent
x figure.
Applying Newton’s second law for translation of mass center, we have
G G
∑ Fi = MaC → mg − T = maC (2)

Applying torque equation for centroidal rotation, we have


G G
∑τ C = ICα → T ( 12 A ) = I Cα

Substituting 1
12
mA2 for IC and α form eq. (1), we have

T = 13 maC (3)

From eq. (2) and (3), we have


Acceleration of the mass center aC = 43 g Ans.

2.3.Friction in Rolling. Kinetic friction opposes the sliding and static friction opposes the tendency of
sliding and at the point of contact. Sliding is apparent so the direction of
kinetic friction. But the tendency of sliding is not apparent so the direction of
static friction. The tendency of sliding is best judged by the direction in which
particles of the body at point of contact would move relative to the surface on
which the body rolls in absence of friction. Determination of direction of static
friction at the point of contact in rolling requires direction of tendency of
sliding. For the purpose solve eq. [8.22] and [8.29] for acceleration aC of the
mass center and angular acceleration α of the body both in absence of friction
at the point of contact and angular velocity. Thereafter, find acceleration of the
particle of the body at the point of contact using relative motion equation. Its
W
aC direction indicates direction of tendency of sliding. However in problems on
rolling, requiring complete solution, it is not necessary to decide direction of
C C aC friction as a priory. A solution of eq. [8.22], [8.29] consistent with conditions of
R
rolling yields correct direction and magnitude of the required static friction.
P
P aC Depending on complexity of the problem it may of may not be necessary to
N solve the above mentioned equations to decide direction of friction and only
qualitative interpretations of these equations would suffice. Consider a bicycle,
Fig. 8.21 (a)
24

which is being paddled and brakes are released. Certainly it is accelerating


forward and both the wheels have angular acceleration in clockwise sense. We
decide direction of friction on both of the wheels. Since it is desired that the
wheels do not slide, the friction on both the wheels must be static friction. In
absence of friction, forces acting on the front wheel are its weight W, the
normal reaction from the ground N and reaction R at the axel from rest of the
body of the bicycle. These forces can not create any torque about the axel but
W G G G the horizontal component of R would set a pure translation motion causing all
aC = α × rC / P
particles of the wheel to accelerate in the forward direction with acceleration
C
C a
C
same as that of its center. Therefore particle P at the point of contact would
R
slide forward relative to the ground in absence of friction as shown in Fig. 8.21
α
(a). This direction of acceleration indicates the tendency of sliding. Thus static
P P
f sf friction (fsf), which opposes the tendency of sliding, must act in the backward
N
direction to keep the point of contact at instantaneous rest and set the wheel
Fig. 8.21 (b) in rolling as shown in Fig. 8.21(b).
On the rear wheel forces applied by the chain driving it crates a torque
abut the axel to rotate the wheel in clockwise sense. In absence of friction
there is no external force in horizontal direction on the
bicycle so the torque would set up pure rotation about the
G aP axel causing the particle of the wheel at the point of
G
τC α τC contact to accelerate in backward direction as shown in
Fig. 8.21 (c). This direction of acceleration indicates the
tendency of sliding of the particle of the wheel at the
P G GP G P fsr point of contact. The force of static friction (fsr), which
aP = α × rP / C
opposes the tendency of sliding, must act in the forward
Fig. 8.21 (c) Fig. 8.21 (d) direction to keep the point of contact at instantaneous
rest and set the wheel in rolling as shown in Fig. 8.21(d).
Following similar analysis we can conclude that
friction force acts in forward direction on powered wheels and in the backward
direction on the wheels which or not powered.

F Example 8.21. A rigid body of mass m and round section of radius r rests on rough
x C horizontal surface. The radius of gyration of the body about its centroidal axis is k. It is
pushed by a constant horizontal force F. The height x of the point where the force is
applied can be adjusted.
(a) Deduce suitable expression for magnitude and direction of the friction force
Fig. Ex. 8.21
necessary to ensure rolling.
(b) How direction of the friction force depends on x.
Solution.
(a) The problem requires solution of the force and the torque equations consistent with
the condition of rolling, so it is not necessary to decide the direction of friction as
y
α ac = α r priory. To start with let the static friction fs acts in the forward direction.
aC
x C
Let the mass center of the body moves towards right with acceleration aC and
angular acceleration α of the body is in clockwise sense as shown in the adjacent
P figure.
Necessary condition for rolling in terms of acceleration aC of mass center and
angular acceleration α of the body is
y

F
x
G G G
aC = α × rC / P →
G
( )
aC = −α kˆ × rjˆ ⇒ aC = α r (1)
mg ΙC α
x maC The forces acting on the body are its weight mg, the normal reaction N from the
C C
ground and the force of static friction fs. These forces are shown in the adjacent
P
figure together with the effective force maC and the effective torque ICα about the
fs
mass center. We analyze the problem as superposition of translation of the mass
N
center and simultaneous centroidal rotation.
25

Applying Newton’s second law for translation of mass center, we have

∑F x = MaCx → F + fs = maC (2)

Applying torque equation for centroidal rotation, we have


G G
∑τ C = ICα → Fx − fsr = I Cα

Substituting mk 2 for IC , and value of α form eq. (1), we have

x mk 2aC
F − fs = (3)
r r2
From eq. (2) and (3), we have

⎛ x − k2 r ⎞
Force of static friction fs = F ⎜ 2 ⎟ Ans.
⎝r+k r⎠

(b) The above expression shows that to ensure rolling

For x > k 2 r , the friction is in direction of the applied force F. ⎫


2 ⎪
For x = k r , no friction is required to ensure rolling. ⎬ Ans.

For x < k 2 r , the friction must be in opposite to the applied force F.⎭

Example 8.22. A uniform rigid body of circular cross section of radius R rolls without
slipping down a slope inclined at an angle θ to the horizontal. The radius of gyration of
C
the body about it central axis of symmetry is k.
(a) Decide nature and direction of the friction force.
θ (b) Derive suitable expressions for the acceleration of mass center of the body.
Fig. Ex. 8.22 (c) What should be the minimum coefficient of friction to ensure rolling?
Solution.
(a) The body rolls down the slope without any slipping; hence there must be static
mg friction between the body and the slope.

fs Forces acting on the body are its weight mg, the normal reaction N from the ground
C
α and the static friction fs. Since its weight and the normal reaction both pass
through the mass center and cannot create any torque, it is the static friction which
P
N must create an anticlockwise torque. Therefore it must act up the plane as shown
θ in the adjacent figure.
(b) The rolling motion is superposition of translation of mass center and simultaneous
centroidal rotation with the following condition between acceleration of mass center
aC, angular acceleration α and the radius r.
G G G
mg sin θ aC = α × rC / P → aC = α r (1)
y x
mg
mg cos θ The free body diagram and effective force maC and the effective torque ICα are
θ ΙCα
C fs
maC
shown in the adjacent figure.
Applying Newton’s second law for translation of mass center, we have
P
N ∑F x = MaCx → mg sin θ − fs = maC (2)

∑F y =0→ N = mg cos θ (3)

Applying torque equation for centroidal rotation, we have


G G
∑ τ C = IC α → fs r = I C α

Substituting mk 2 for IC , and value of α form eq. (1), we have

mk 2
fs = α (4)
r2
From eq. (2) and (4), we have
26

g sin θ
Acceleration of the mass center aC = Ans.
k2
+1
r2
(c) For rolling, the required friction obtained from eq. (2) and (4) is
mg sin θ
fs = (5)
r2
+1
k2

It must satisfy the following equation fs ≤ μs N . Now from eq. (3) and (5), we have

k2
μs ≥ tan θ
k + r2
2

From the above equation, the minimum required coefficient of friction is

k2
μs min = tan θ Ans.
k + r2
2

Example 8.23. A block of mass m is attached at one end of a thin light cord, which
passes over an ideal pulley. At the other end, it is wrapped around a cylinder of mass
M, which can roll without slipping over a horizontal plane.
m
(a) What is the acceleration of the block?
Fig. Ex. 8.23 (b) What is the friction force on the cylinder?
Solution. The problem requires solution of the force and the torque equations
consistent with the condition of rolling, so it is not necessary to decide the direction of
friction as priory. To start with let the static friction fs acts in the forward direction.
y A a
(a) Let the block descend with acceleration a. Since the cord is inextensible the top
aC
x
C
point A of the cylinder also moves with the same acceleration. Applying relative
α motion equation with the condition required for rolling that the particle of the
cylinder at the point of contact has no acceleration parallel to the horizontal plane.
P
a
G G G
a A = α × rA / P → ( ) ( )
a A iˆ = −α kˆ × 2rjˆ ⇒ α =
a
2r
(1)

From eq. (1) and relative motion equation for P and the center C, we have
G G G
aC = α × rC / P → ( ) ( )
aC iˆ = −α kˆ × rjˆ ⇒ aC = 12 a (2)

T The block is in translation motion under the action of its weight mg and tension T
in the string. These forces and the effective force ma are shown in the adjacent
figure.
mg ma
Applying Newton’s second law for translation of mass center, we have
G G
∑ Fi = MaC → mg − T = ma (3)

y
The cylinder is in rolling under the action of its weight Mg, normal reaction N form
Mg T the ground; tension T in the cord and force of static friction fs. These forces, the
x
I Cα Ma effective force MaC and the effective torque ICα are shown in the adjacent figure.
C
C
P C Applying Newton’s second law for translation of mass center, we have

N
fs
∑F x = MaCx → T + fs = MaC

Substituting aC from eq. (2), we have

T + fs = 12 Ma (4)

Applying torque equation for centroidal rotation, we have


G G
∑ τ C = IC α → Tr − fs r = I C α

Substituting 1
2
mr 2 for IC , and value of α form eq. (1), we have

T − fs = 14 Ma (5)
27

From eq. (3), (4) and (5), we have


8mg
Acceleration of the block a= Ans.
3M + 8m

(b) From eq. (4), (5) and above value of acceleration a, we have
Mmg
Force of static friction fs = → Ans.
3M + 8m

Example 8.24. A uniform sphere of mass m rests on a rough plank, which can slide on
a smooth horizontal surface. The plank is made to move with constant acceleration ao to
the right as shown in the figure. The friction is sufficient enough to prevent sphere
ao
from sliding on the plank. Find the Acceleration mass center of the sphere and friction
force acting on it.
Fig. Ex. 8.24
Solution. The point of contact has tendency to slide backwards relative to the plank,
therefore force of static friction fs acts towards the right on the sphere.

y
Since the sphere cannot slide it rolls rotating anticlockwise on the plank. The point of
α a contact P moves with the same horizontal component of acceleration ao at which plank
x
is moving. Let angular acceleration of the sphere is α, acceleration of its mass center C
C
C
aP = ao is aC and radius of the sphere is r. These quantities are shown in the adjacent figure.
ao
P
Applying relative motion equation, we have
G G G
aC = aP + aC / P → aC iˆ = ao iˆ + α kˆ × rjˆ ⇒ aC = ao − α r (1)

y
The forces acting on the sphere are its weight mg, the normal reaction N form the
mg
plank and the force of static friction fs. These forces, the effective force maC and the
x
maC effective torque ICα are shown in the adjacent figure.
C C
ΙCα Applying Newton’s second law for translation of mass center, we have
fs
N ∑F x = MaCx → fs = maC (2)

Applying torque equation for centroidal rotation, we have


G G
∑ τ C = IC α → fs r = I C α

Substituting 2
5
mr 2 for IC , and value of α form eq. (1), we have

fs = 25 mao − 25 maC (3)

From eq. (2) and (3), we have


Acceleration of mass center aC = 72 ao Ans.

Force of static friction fs = 72 mao Ans.

3. Energy Methods. Newton’s laws of motion tell us what is happening at an instant, while method
of work and energy equips us to analyze what happens when a body moves
from on place to other or a system changes its configuration. In chapter 5,
energy methods have been introduced for bodies in translation motion
assuming them particles. In this section, we introduce how to use methods of
work and energy to analyze motion of rigid bodies. First we discus the concepts
of work, potential energy and kinetic energy as interpreted for rotation motion
then we discuss the principle of work and energy to analyze rotation of rigid
bodies and finally we discuss conservation mechanical energy in rotation
motion.
28

In section 5.1, work of a force was defined as the scalar product of the force
3.1. Concept of Work in
rotation motion. vector and displacement vector of the point of application of the force.
G
If during the action of a force F its point of application moves from position
G G
r1 to r2 , the work W1→ 2 done by the force is expressed by

r2 G
G
G
W1→2 = ∫G F ⋅ dr 5.2
r1

For calculation of work in rotation motion too, we use the above equations.
G Q However in fixed axis rotation the above equation reduces to another equation
F
relating torque and angular displacement. Consider a rigid body capable of
G
P rotating about a fixed axis through point O. Let a force F act on a particle P of
G θ
the body. During continuous action of the force the body rotates through angle
r
ω θ and the particle reaches position Q following a circular path of radius r as
G G G
O shown in Fig. 8.22. At the instant shown the torque vector τ o = r × F and the
G
angular displacement vector dθ in an infinitesimally small time interval dt
Fig. 8.22 both point perpendicularly into the plane of rotation. The work done dW by the
force during the time interval dt equals to the scalar product of the force vector
G G
and the displacement dθ × r of the particle.
G G G
(
dW = F ⋅ dθ × r )
G G G G
Since scalar triple product a ⋅ (b × c ) = b ⋅ ( c × a ) = ( c × a ) ⋅ b , the above equation
G G G G G

reduces to
G G
dW = τ o ⋅ dθ [8.31]

The work done by the torque during a finite rotation of the rigid body from
initial value θi of the angle θ to final value θf, can be obtained by integrating
both the sides of the above equation.
θf G G
Wi → f = ∫ τ o ⋅ dθ [8.32]
θi

F Example 8.25. A thin light cord is wound around a uniform cylinder placed on a rough
horizontal ground. When free end of the cord is pulled by a constant force F the cylinder
rolls. Denote radius of the cylinder by r and obtain expression for work done by each of
the forces acting on the cylinder when center of the cylinder shifts by distance x.
Fig. Ex. 8.25
Solution. Forces acting on the cylinder are its weight W, the normal reaction from
the ground N, the tension T in the cord and the force of static friction fs. The tension in
mg F mg F the cord equals to the applied force F. These forces are shown in the adjacent figure.
A A
In rolling point of contact P is at instantaneous rest, the center C moves with velocity
C x C
vC = ω r and the top point moves with velocity vA = 2vC = 2ω r both parallel to the
P P
fs fs
surface on which body rolls. Since the cord is inextensible displacement of the top point
equals to the displacement of the free end of the cord. These fact suggests that during
N N
displacement x of the center the free end of the cord shift through a distance 2x.
Work done by the weight of the cylinder.
Wg = 0 The weigh is assumed to act on the center of gravity which coincides with
the mass center in uniform gravitation field near the ground. The
displacement x of the mass center and weight both are perpendicular to
each other so the work done by gravity is zero.
Work done by the normal reaction of the cylinder
WN = 0 The normal reaction acts on the particle of the body which is in contact
with the ground. The particles making contact continuously change and
29

remain at instantaneous rest during contact. Therefore normal reaction


does no work.
Work done by the force of static friction.
Wfs = 0 The force of static friction fs acts on the particle of the body which is in
contact with the ground. The particles making contact continuously change
and remain at instantaneous rest during contact. Therefore force of static
friction fs does no work.

Work done by the tension in the cord.


WT = WF The particle of the wheel on which the tension in the cord acts is at the top
point. Though this particle is also continuously changing but it is not in
vA dt instantaneous rest and has velocity vA. So in every infinitesimally small
T =F
A time interval displacement of this particle is vA dt = 2vC dt , thus work done
dWT by the tension during a time interval dt

dWT = T ( 2vC dt ) = 2 F (vC dt )

When the center shift by a distance x the work one by the tension becomes
WT = WF = 2 Fx

3.2.Potential Energy of a Since potential energy of a system is function of its configuration and does not
rigid body. depend on the manner in which the system is brought into a particular
configuration, hence it does not depends on motion involved whether it is
translation, rotational or their combination. Thus potential energy functions
which we have developed in chapter 5, will remain unchanged.

3.3.Kinetic Energy of a A rigid body can be represented as a system of large number of particles,
rigid body in rotation which keep their mutual distances unchanged in all circumstances. Kinetic
motion. energy of the whole body must be sum of kinetic energies of all of its particles.
In this section we develop expressions for kinetic energy of a rigid body.

Kinetic Energy of a rigid In this section we learn how to express kinetic energy of a body in plane
body in plane motion. motion. Till so far we have learnt that equations of kinematics and torque
equation of a body in plane motion can be expressed by representing the plane
motion as superposition of translation of its mass center and simultaneous
centroidal rotation. Now we inquire whether the kinetic energy too can be
represented as superposition of translation of mass center and simultaneous
centroidal rotation.

Superposition of translation of mass center and simultaneous Centroidal


Rotation. Consider the rigid body of mass M as described in Fig. 8.13. It is in
plain motion i.e. rotation motion about an axis, which is in translation motion.
Since all particles of the body move in planes parallel to the x-y plane, every
G
G
vC section of the body parallel to the x-y plane are in similar kind of motion and
y vi / C
can be used to represent plain motion of the body. In the Fig. 8.23 a section in
G
mi vi the x-y plane of a body in plain motion is shown. The plain motion of the body
G
can be represented as superposition of translation of its mass center C and
vC simultaneous centroidal rotation. Consider a particle A of mass mi moving
C G G
with velocity vi . The body is rotating with angular velocity ω in the clockwise
G
sense and its mass center is moving with velocity vC relative to an inertial
O x frame represented by a coordinate system in the figure. Velocity of the particle
Fig. 8.23
can be represented by the following equation.
30

G G G G G G
vi = vC + vi / C = vC + ω × ri / C [8.33]

The kinetic energy of the whole body relative to the inertial frame is the
sum of kinetic energies of all such particles forming the body, thus we have

K =∑ ( 1
2
mi vi2 )
G
Substituting vi from eq. [8.33] in the above equation we have

∑{ }
G G G 2
K = 1
2
mi (vC + ω × ri / C )

K = 1
2 ( ∑ m )v i
2
C + 1
2 ( ∑m r )ω
2
i i/C
2 G G
{ G
}
+ vC ⋅ ω × ( ∑ mi ri / C )

The second term contains ∑m r 2


i i /C , which equals to moment of inertia IC of
the body about centroidal axis. The last term vanishes because it contains sum
of mass moments in center of mass frame, which reduces to zero i.e.
G
∑ miri / C = 0 . Now we can express the kinetic energy of the rigid body by the
following equation.
K = 12 MvC2 + 12 IC ω 2 [8.34]

The first term in the right hand side is the kinetic energy due to
translation of the mass center and the second term is the kinetic energy due to
the centroidal rotation. It suggests that the kinetic energy of a rigid body
equals to the sum of kinetic energy due to translation of the mass center and
kinetic energy due to pure rotation about the mass center. These two terms of
eq. [8.34] are sometimes known as translational kinetic energy and rotational
kinetic energy respectively. The kinetic energy due to translation of the mass
center of a body cannot be changed without any external force. The internal
forces arising within the body can only change its kinetic energy due to
rotation about the mass center.

Rotation about the instantaneous axis of rotation. Kinetic energy of a


y
G rigid body in plane motion can also be expressed in a single term of kinetic
vi
energy due to rotation about the instantaneous axis of rotation. Consider a
mi G
G
ri / C G
body in plane motion represented in Fig. 8.24, in which velocity vector vi of a
vC G
particle mi and vC of the mass center C relative to the coordinate system are
G C
ri / P G
rC / P shown. If the point P is the ICR, the instantaneous axis of rotation passes
G
G through it and is perpendicular to the x-y plane. The position vectors ri / P and
P ω G
rC / P must the perpendicular to the corresponding velocity vectors. The velocity
G G G
vectors of the particle mi and mass center bear the relations vi = ω × ri / P and
O G G G G G G
x
vC = ω × rC / P respectively. Substituting the expression vC = ω × rC / P in eq. [8.34],
Fig. 8.24
we have
G G
K = 12 M (ω × rC / P ) + 12 I C ω 2
2

G G G G
Since ω and rC / P are perpendicular, we can substitute vC = ω × rC / P = ω rC / P in
the above equation. After this substitution the above equation reduces to

K= 1
2 ( Mr ) ω 2
C/P
2
+ 12 IC ω 2 = 1
2 ( Mr 2
C/P )
+ IC ω 2

The centroidal axes and the instantaneous axis of rotation both are parallel to
G
each other and the distance between them is rC / P , therefore from the theorem
of parallel axis, the moment of inertial IP of the body about the instantaneous
axis of rotation must be I P = I C + MrC2 / P . So we can write

K = 12 I P ω 2 [8.35]
31

Kinetic energy of a rigid body in plane motion can be expressed in a single


term of kinetic energy due to rotation about the instantaneous axis of rotation.

Kinetic Energy in fixed axis rotation. The instantaneous axis of rotation is


at instantaneous rest. If we assume the body described in Fig. 8.24 rotating
about a fixed axis through point P, eq. [8.34] and [8.35] can be used to express
its kinetic energy.

3.4 Power. In section 5.4, power was defined as the time rate at which work is done and
takes into account the duration in which work is done. It was defined by eq.
5.6.
dW dK
P= = 5.6
dt dt
G
In case of a body in translation motion moving with velocity v under the
G
action of a force F , the instantaneous power p was expressed by the following
equation.
dW d G G G G
P= = F ⋅ dr = F ⋅ v 5.7
dt dt
If a rigid body is in rotation motion, power can be calculated by the equation
G
5.6. For a rigid body is rotation at angular velocity ω in fixed axis rotation
G
under the action of torque τ , the eq. 5.6, yields to the following equation.
dW d G G G G
P= = τ ⋅ dθ = τ ⋅ ω [8.36]
dt dt

ω
O Example 8.26. A rod of mass m and length A is pivoted to a fixed support at one of its
ends O. It is rotating with constant angular velocity ω. Write expression for its kinetic
Fig. Ex. 8.26
energy.
Solution. If the point C is the mass center of the rod, from theorem of parallel axes,
1
2
A the moment of inertia IO of the rod about the fixed axis is
O
C
ω I O = I C + m (OC ) → I O = I C + 14 mA2
2

Substituting 1
12
mA2 for IC, we have

I O = 13 mA2

Kinetic energy of the rod equals to kinetic energy due to rotation about the fixed axis.

K = 12 I o ω 2 → Using above expression for IO, we have

K = 16 mA2 ω 2 Ans.

Example 8.27. A uniform rigid body of mass m and round section of radius r is rolling
ω on horizontal ground with angular velocity ω. Its radius of gyration about the
C centroidal axis is k.
(a) Write expression of its kinetic energy.
(b) Also express the kinetic energy as sum of kinetic energy due to translation of mass
Fig. Ex. 8.27 center and kinetic energy due to simultaneous centroidal rotation.
Solution.
(a) The point of contact with ground of a body rolling on the ground is its ICR. Let the
point P is the ICR as shown in the adjacent figure. The geometrical center C of a
32

uniform body and the mass center coincide. Therefore moment of inertia IP of the
vC = ωr body about the ICR can be written by using the theorem of parallel axes.
C
I P = I C + m ( PC ) →
2
ω I P = I C + mr 2

P Substituting I C = mk2 , we have

(
I P = m k2 + r 2 ) (1)

Kinetic energy of a rigid body equals to kinetic energy due to rotation about the
ICR.

K = 12 I P ω 2 → Substituting IP from eq. (1), we have

(
K = 12 m k2 + r 2 ω 2 ) Ans.

(b) Kinetic energy of the body also equals to sum of kinetic energy due to translation of
its mass center and kinetic energy due to simultaneous centroidal rotation.

K = 12 mvC2 + 12 I C ω 2 → Substituting condition for rolling vC = ω r and I C = mk2 ,


we have

( )
K = 12 m (ω r ) + 12 mk2 ω 2 = 12 m r 2 + k2 ω 2
2
Ans.

3.5.Work and Energy Work energy theorem can be applied in similar fashion as it was applied to
Theorem. analyze translation motion of a single body or a system of several bodies.
The work energy theorem relates kinetic energy K1 and K2 of a body in its
initial and final position with work W1→2 done by all the external forces acting
on the body to carry it form the initial position to the final position according
to the following equation.
W1→2 = K 2 − K1 [8.37]

This equation is applicable in all inertial as well as noninertial frames.


While writing the equation from any inertial frame velocities of the body in the
initial and the final positions relative to the frame under consideration must
be used to express kinetic energies and displacement of the body relative to the
frame under consideration must be used to calculate the work done by all the
forces. If the frame under consideration is a noninertial one, in addition to
considering velocities and displacement relative to the frame under
consideration for expressing kinetic energies and work done, work done by
pseudo forces must also be considered.
For a system of several bodies the corresponding equation of work energy
theorem can be obtained by applying the theorem for each individual body and
then adding all of them. In this way we obtain an equation of the following
form.

∑W 1→ 2 = ∑ K 2 − ∑ K1 [8.38]

Here the term ∑ W1→2 equals to the work of all the forces acting on various
bodies irrespective of whether the force are internal or external from point of
view of the system under consideration. The terms ∑ K1 and ∑ K 2 are the sum
of kinetic energies of all the bodies in initial and final configurations of the
system.
In systems of several bodies interconnected by links of constant length e.g.
inextensible cords, rods etc or body in direct contact the total work of internal
forces vanishes, because internal forces occurs in pairs of equal and opposite
forces and in all these cases points of applications of any pair of internal force
33

moves through equal displacement in every infinitely small time interval. This
fact forms the basis of principle of virtual work discussed in chapter 3 article
5.1.

3.6. Conservation of The work of conservative forces equals to decrease in potential energy. When a
Mechanical Energy. single rigid body moves or a system of rigid body changes its configuration
under the action of conservative forces and nonconservative forces are either
not present or if present do no work, the work energy principle can be
expressed as
W1→2 = U1 − U 2 = K 2 − K1

U1 + K 2 = U 2 + K 2 [8.39]

The above equation expresses the law of conservation of mechanical energy


and states that if a rigid body moves or a system consisting of several rigid
bodies changes its configuration under action of conservative forces the
mechanical energy i.e. sum of kinetic and potential energy remain constant;
provided that nonconservative, if present, do no work,
Though the work energy principle and the law of conservation of
mechanical energy are equivalent, we prefer to use the former for its more
general nature to account for nonconservative forces.

A O C B Example 8.28. A uniform rod AB of mass m and length A is pivoted at a point (O) to
rotate in the vertical plane. The rod is held in horizontal position and released. Find
x the distance x of the pivot from the mass center (C) of the rod, so that angular speed ω
Fig. Ex. 8.28 of the rod as it passes through the vertical position is maximum.
Solution. The problem involves change in angular velocity with change in position,
therefore demands application of principle of work and energy.
A Initial
Position The rod when released rotates about a fixed horizontal axis passing through the point
OO
O. Its initial and final positions are shown in the adjacent figure.
x Moment of inertia of the rod about the pivot O can be calculated by theorem of parallel
C axes.

I o = I C + mx 2 → Substituting 1
12
mA2 for IC, we have
Initial
Position B
(
I o = 121 m A2 + 12x 2 ) (1)

Kinetic energy in the initial position.

K = 12 I oω 2 → K1 = 0 (2)

Kinetic energy in the final position.

K = 12 I oω 2 → Substituting for Io form eq. (1), we have

K2 = 1
24 (
m A2 + 12x 2 ω 2 ) (3)

Only gravity does work when the rod moves from the initial to final position.
G G
W = ∫ F ⋅ dr → W1→2 = mgx (4)

Substituting values form eq. (2), (3) and (4) in equation of work energy principle, we
have

W1→2 = K 2 − K1 → ω = 24 gx ( A2 + 12x 2 ) (5)

The above equation expresses angular velocity of the rod when it passes the vertical
position. For it to be maximum
34

dω A
=0→ x= Ans.
dx 12

Example 8.29. A uniform rigid body mass m and round section of radius r rolls down a
C slope inclined at an angle θ to the horizontal. The radius of gyration of the body about it
central axis of symmetry is k.
x
C (a) Derive suitable expressions for angular velocity and velocity of its mass center after
G
vC ω it covers a distance x.
θ
(b) Obtain expression for its angular acceleration and acceleration of the mass center.
Fig. Ex. 8.28 Solution. The problem involves change in angular velocity with change in position,
therefore demands application of principle of work and energy
The geometrical center and mass center for uniform bodies coincide; therefore center C
is the mass center.
In rolling the point of contact P must always be at instantaneous rest and angular
velocity ω, velocity of center C, angular acceleration α and acceleration of the center
must bear the following relations.
vC = ω r and aC = α r (1)

(a) The rolling motion can be analyzed as superposition of translation of the mass
center and simultaneous centroidal rotation.
Kinetic energy in the initial position.
K1 = 0 (2)

Kinetic energy in the final position.

K = 12 mvC2 + 12 I Cω 2 → Substituting for I C = mk2 and vC form eq. (1), we have

(
K 2 = 12 m r 2 + k2 ω 2 ) (3)
mg sin θ
The forces acting on the body are its weight mg, the normal reaction N from the
E
mg cos θ slope and the force of static friction fs. These forces and displacement are shown in
θ C the adjacent figure. The normal reaction and the force of static friction do no work
in rolling, it the weight, which does work.
x fs G G
W = ∫ F ⋅ dr → W1→2 = mgx sin θ (4)
N
Substituting values form eq. (2), (3) and (4) in equation of work energy principle,
θ
we have

2 gx sin θ
W1→2 = K 2 − K1 → ω= Ans.
k2 + r 2
Substituting vC from eq. (1), we have

2 gx sin θ
vC = Ans.
(k 2
)
r2 + 1

(b) Acceleration a, velocity v and position coordinate x bear the relation a = v ( dv dx ) .


Therefore acceleration of mass center of the body.
dvC g sin θ
aC = vC → aC = Ans.
dx (k 2
)
r2 + 1

Substituting aC in eq. (1), we have the angular acceleration of the body.

g sin θ
aC = α r → a= Ans.
(k r) + r
2

ℜ The previous example represents an alternative way to find acceleration and


angular acceleration by using principle of work and energy. After enough
35

practice you will be able to write equation of work energy principle for
majority of situation by inspection. In such problems this method yields
simpler solution as compared to application of Newton’s laws of motion.

O Example 8.30. A homogeneous sphere of radius r is released from angular position θ


θ as shown in the Fig. Ex. 8.30 to roll on a cylindrical track of radius R. The center of the
track is at O. Find velocity of its mass center and the normal reaction from the track
R when it passed the bottom of the track. Assume sufficient friction to prevent sliding
anywhere during the motion.
Solution. When the sphere rolls down the track its mass center C moves on the
circular path of radius R − r shown by dashed line in the adjacent figure. If we denote
Fig. Ex. 8.30 velocity of its center by vC and angular velocity by ω, the condition of rolling suggest us
to write
vC = ω r (1)

The problem involves velocity change with change in location, so we can use work
energy principle.
O The sphere starts from rest so its kinetic energy in the initial position is zero.
θ
K = 12 I ω 2 → K1 = 0 (2)
R
In the final position it is at the bottom, let in this position denote velocity of its center
vC = ωr
by vC and angular velocity by ω as shown in the adjacent figure. It is in rolling so its
P
ω
kinetic energy at the final position can be written as in the following equation.

K = 12 I ω 2 → K 2 = 12 I P ω 2

Substituting I P = 75 mr 2 from theorem of parallel axes and


vC = ω r from eq. (1), we have

K2 = 7
10
mvC2 (3)

During its rolling forces acting on the sphere are its weight, normal reaction and force
of static friction from the ground. The normal reaction and the force of static friction act
always on the point of contact which is always at instantaneous rest do these two force
do no work. It is the weight of the sphere which does work and constitute the total work
W1→2 by all the forces.
G G
W = ∫ F ⋅ dr → W1→2 = mg ( R − r )(1 − cos θ ) (4)

Substituting values form eq. (2), (3) and (4) in equation of work energy principle, we
have

10
W1→2 = K 2 − K1 → vc = g ( R − r ) (1 − cos θ o ) Ans.
7

O O
To calculate the normal reaction by the track at the bottom point we have to use
n
Newton’s laws of motion. At the bottom point its velocity and angular velocity both
acquire their maximum values so the tangential acceleration and the angular
τ acceleration of the sphere are zero. It requires no net tangential force, therefore friction
mg mvC2
when the sphere is at the bottom is zero. Since the mass center is in circular motion, it
R−r
must have only a normal component of acceleration acting toward the center O. The
forces acting on the sphere when it is at the bottom of the track, and the effective force
(centripetal force) is shown in the adjacent figure.
N
Applying Newton’s second law for translation of mass center, we have

mvC2
∑F n = MaCn → N − mg =
R−r
(5)

Substituting value of vC, we have

mg
N =
7
(17 − 10 cos θo ) Ans.
36

Example 8.31. Axel of a cylinder of mass m and radius r is attached to the free end of
a spring of force constant k. The other end of the spring is fixed as shown in the Fig. Ex.
8.31. The cylinder is displaced from the mean position by an amount xo and released. If
C
it rolls on the horizontal surface, find its angular velocity as function of its distance x
from the mean position.
Solution. There is no need to decide kinematical relations.
Fig. Ex. 8.31
The problem involves velocity change with change in location, so we can use work
energy principle.
xo
Its kinetic energy K1, in the initial position, when it was at distance xO form the mean
vC C position is zero.
ω
K = 12 I ω 2 → K1 = 0 (1)
P
In the final position, when it is at distance x from the mean position it is rolling. Its
kinetic energy K2 is given by the following equation.

K = 12 I ω 2 → K 2 = 12 I P ω 2

Substituting I P = 32 mr 2 from the theorem of parallel axes, we


have

K 2 = 32 mr 2 ω 2 (2)

The forces acting on the cylinder are its weight, the spring force at the axel, normal
reaction and force of static friction from the ground. Displacement of the mass center is
normal to the weight, therefore weight does no work. The normal reaction and the force
of static friction both act at the point of contact P, which is always at instantaneous
rest, therefore they also do no work. It is the spring force which does work and
constitute the work W1→2 by all the forces in carrying the cylinder from initial to final
position. Being a conservative force its work can be calculated as difference in potential
energy stored in it.

W1→2 = U1 − U2 → Substituting U = 12 kx 2 , we have

(
W1→2 = 12 k x o2 − x 2 ) (3)

Substituting values form eq. (1), (2) and (3) in equation of work energy principle, we
have

W1→2 = K 2 − K1 → ω=
(
k x o2 − x 2 ) Ans.
3r 2

Example 8.32. A uniform sphere of mass m and radius r is placed on top of a half
cylinder of radius 2r which is fixed to the ground as shown in Fig. Ex 8.32. When the
P
sphere is displaced gently, it begins to roll down the surface of cylinder. If the
θ coefficient of sliding friction is 1/3, find the value of angle θ at which sliding begins.
Solution. Position where sliding beings, we have to apply Newton’s laws of motion.
Till the moment sliding begins the sphere is rolling without sliding, so its mass center
Fig. Ex. 8.32
velocity v, angular velocity ω, mass center tangential acceleration aτ and angular
acceleration α bear the following relations.

v = ωr ; and aτ = α r (1)
mg sin θ The forces acting on the sphere are its weight mg, the normal reaction N from the
mg sphere, and the force of static friction. These forces and normal and tangential
ICα
fs
mg cos θ components man and maτ of the effective forces and the effective torque ICα are shown
in the adjacent figure.
maτ
N man Applying Newton’s second law of motion, we have
θ n τ
∑F n = man → mg cos θ − N = man
37

Substituting an = v2 ( R + r ) , we have

mv2
mg cos θ − N = (2)
R+r

∑ Fτ = maτ → mg sin θ − fs = maτ (3)

Applying torque equation, we have

∑τ C = I Cα → fsr = 25 mr 2α

Substituting aτ = α r from eq. (1), we have

fs = 52 maτ (4)

From eq. (3) and (4), we have


fs = 72 mg sin θ (5)

At the angular position where sliding begins, friction will have its limiting value.
fs = μ N = 13 N (6)

This condition requires to evaluate N, which we can obtain from eq. (2). It needs to
evaluate speed at the angular position θ. It involves velocity change with position
change and demand use of work energy principle.
Kinetic energy in the initial position (top), where the sphere started is zero.

K = 12 I ω 2 → K1 = 0 (7)

The final position is at angle θ, where the sphere is rolling, so its kinetic energy is

K = 12 I P ω 2 → K2 = 1
2 ( 7
5 )
mr 2 ω 2

Substituting v = ω r from eq. (1), we have

K 2 = 107 mv2 (8)

During its rolling from the top to angular position θ, only weight of the sphere does
work. It is given by the following equation.
G G
W = ∫ F ⋅ dr → W1→2 = mg ( R + r )(1 − cos θ ) (9)

Substituting values form eq. (7), (8) and (9), into the equation of work energy principle,
we have

mv2 10
W1→2 = K 2 − K1 → = mg (1 − cos θ ) (10)
R+r 7
Obtaining N form eq. (2) and (10) and fs form (5), substitute into (6), we have

θ = 37° Ans.
Example 8.33. A light inextensible cord ABCDE is wound over the disk-A, which can
m A
rotate about a fixed horizontal axis through its center. The cord passes underneath
Disk B another identical disk-B and the fixed to ceiling as shown Fig. Ex. 8.33. The disk-B is
Disk A held keeping the system at rest. When released it descends. Determine the angular
C
D B velocity and velocity of the center C of the disk-B when the disk-B descends a height h.
Solution. The disk-A rotates in anticlockwise sense un-wrapping the cord at point A.
Fig. Ex. 8.33
Since the cord is inextensible the point B on the cord also descends with the same
instantaneous velocity at the point A. If we denote angular velocity of the disk-A by ω1,
we can write
vA = vB = ω1r (1)

ω1
The point E is always at rest and because the cord is inextensible, the point D which is
A
in contact with the disk-B remains always at instantaneous rest. Therefore motion of
E
disk-B resembles rolling on a vertical stationary surface ED. If we denote angular
Disk B velocity of the disk-B by ω2, we can write
Disk A
C
D ω2 B
vC
vB = 2vC
38

vC = ω2r (2)

vB = 2ω2r (3)

From eq. (1) and (2), we have


ω1 = 2ω2 (4)

These velocities are shown in the adjacent figure.


Both the disks start from rest so kinetic energy of the system in the initial
configuration is zero.
K1 = K1 A + K1 B → K1 = 0 (5)

In final configuration disk-1 is in rotation about its fixed axis and disk-2 is in a motion
resembling rolling on vertical surface ED.

A
K 2 = K2 A + K 2B → K 2 = 12 I ω12 + 12 I Dω22
E
T2
Substituting I = 12 mr 2 , I D = 32 mr 2 and ω1 from eq. (4), we have
T1

T2 K 2 = 74 mr 2ω22 (6)
T1
C
D B The various forces acting on the disks are shown in the adjacent figure. Here the
reaction of axel and weight of the disk-A are not shown as they only try to keep the disk
in translational equilibrium and do no work. The point E is always at rest and D is
mg
always at instantaneous rest so neither of the tension forces T2 do any work. The
h displacements of point A and B in every infinitely small time interval are equal,
therefore in every infinitely small time interval the tension force T1 does same amount
of positive work on disk-A as its reaction does negative work on the disk-B. Thus total
work done by pair of these tension forces is zero. The only force which does work is the
weight of the disk-B. Therefore we write
G G
W = ∫ F ⋅ dr → W1→2 = mgh (7)

Substituting values form eq. (5), (6) and (7), into the equation of work energy principle,
we have

4 gh
W1→2 = K 2 − K1 → ω2 = Ans.
7r 2

4 gh
From eq. (1), we have vC = Ans.
7

F
P Example 8.34. A plank P mass M is supported by two uniform identical cylinders,
which rest on horizontal ground. Each of the cylinders is of man m and radius r. When
the plank is pulled horizontally with a constant force F the cylinders roll without
sliding on the ground as well as at points where they make contact with the plank.
Assume the plank is long enough.
Fig. Ex. 8.34
(a) Find the velocity of the plank after it has been displaced by a horizontal distance x.
(b) Use the expression obtained to find acceleration of the plank.
Solution.
(a) Distance between any two points on the plank cannot change so velocities of its
points where it makes contact with the cylinders always remain equal. Thus
velocities of points A and B always remain equal.
In rolling on a stationary surface velocity of the top point of top point of a round
vP vP P section body is double of that of its center and velocities of top points of both the
A B cylinders are always equal, therefore centre of both the cylinders move with the
vC vC
CA CB same velocity that is half of the velocity of the plank. Both the cylinders are in
ω ω
identical motion. The velocity of the plank vP, velocities of centers of the cylinders
D E
vC and angular velocities ω of the cylinders are shown in the adjacent figure. We
can write
vC = ω r = 12 vP (1)
39

The system starts from rest, therefore kinetic energy K1 of the system in the initial
configuration is
K1 = K A1 + K B1 + K P1 → K1 = 0 + 0 + 0 = 0 (2)

In the final configuration the plank is in translation motion with velocity vP and the
cylinders are rolling with angular ω velocity given by eq. (1). The kinetic energy of
the system in the final configuration is

K2 = K A2 + K B2 + K P 2 → K 2 = 12 MvP2 + 12 I D ω 2 + 12 I E ω 2

Substituting I D = I E = 32 mr 2 and ω = vP ( 2r ) from eq.


(1), we have

⎛ 4 M + 3m ⎞ 2
K2 = ⎜ ⎟ vP (3)
⎝ 8 ⎠
Forces acting on the various bodies are their weights; normal reactions form the
ground on both the cylinders, the normal reaction between the cylinders and the
plank, force of static friction between the cylinders and the plank and force of static
friction between the cylinders and the ground. Displacements of the bodies are
horizontal so none of weight does work. Normal reactions and forces of static
friction from the ground all act on points D and E which are always at
instantaneous rest and hence do no work. Normal reactions and forces of static
friction between the cylinders and the plank constitute action reaction pairs of
internal forces and do a total of zero work. Thus net work is done by the applied
force F pulling the plank. We can write
G G
W = ∫ F ⋅ dr → W1→2 = Fx (4)

Substituting values form eq. (2), (3) and (4), into the equation of work energy
principle, we have

8 Fx
W1→2 = K 2 − K1 → vP = Ans.
4 M + 3m

(b) Acceleration a, velocity v and position coordinate x bear the relation a = v ( dv dx ) .

4F
Acceleration of plank is a= Ans.
4M + 3m

4. Angular Momentum Newton’s laws of motion tell us what is happening at an instant, methods of
and Impulse. work and energy equip us to analyze velocity change with space interval, while
methods of impulse and momentum equip us to analyze velocity change during
a time interval or very short duration events. In this section we discuss
concept of angular impulse, angular momentum of rigid body, angular impulse
momentum principle and conservation of angular momentum.

4.1.Angular Impulse. Effect of a force cannot be decided, if we know only its magnitude, direction
and point of application but duration of its action must also be known. Like
impulse of a force angular impulse of a constant torque equals to product of
the torque and concerned time interval and if the torque is not constant it
must be integrated with time over the concerned time interval.
G
If torque τ o about an axis passing through O is constant, its angular impulse
G
during a time interval from t1 to t2 denoted by J o,1→2 is given by the following
equation.
G G
J o ,1→2 = τ o (t2 − t1 ) [8.40]
40

G
If torque τ o about an axis passing through O is time varying, its angular
G
impulse during a time interval from t1 to t2 denoted by J o,1→2 is given by the
following equation.
G t2 G
J o ,1→2 = ∫ τ o dt [8.40]
t1

4.2.Angular Momentum of Like linear momentum angular momentum of a rigid body is a vector quantity,
a Rigid Body. which provides us with a measure of amount of rotation motion. The angular
momentum of a particle about an origin is the moment of its linear momentum
i.e. cross product of its position vector of the particle with its linear
momentum. The angular momentum of a system of particles is the sum of
angular momentum all the particles within the system. A rigid body is an
assemblage of large number of particles maintaining their mutual distances
intact under all circumstances, therefore angular momentum of a rigid boy
about an axis must be sum of angular momentum of all of its particles.

Angular Momentum about Imagine a rigid body rotating with angular velocity ω about an axis coinciding
a point and about an axis. with the z-axis as shown in Fig. 8.25. Its angular momentum about the origin
equals to the sum of angular moments of all of its particles about the origin.
Consider an infinitely small portion of mass dm at P of the rigid body. It moves
on circular path of radius r parallel to the x-y plane with speed v = ωr . The
center C of the circular path is on the z-axis. Let its coordinates of C are (0, 0,
G
z). The angular momentum dLO of the portion dm about the origin O is given
by the following equation.
G G G
z dLO = rP × dmv [8.41]
G G
ω
dLo This angular momentum vector can be resolved into two components
G G
dLz along the axis of rotation and dLxy parallel to the x-y plane.
G G G G
G
dLxy
dLz Substituting v = ω × r in eq. [8.41] and simplifying vector triple
G G G G G G G G G
product term rP × (ω × r ) , by using property a × (b × c ) = ( a ⋅ c ) b − ( a ⋅ b ) c ,
G G G G
r
C
dmv = dmωr
x we can write
P
G G G G G G G G G G
G dLO = rP × (ω × r ) dm = ( rP ⋅ r ) ωdm − ( rP ⋅ ω ) rdm
rP
O G G
y
Substituting rP = zkˆ + r in the above equation, we have
x
G G G
dLO = ωr 2 dm − zω rdm
Fig. 8.25 Angular momentum about the
origin of a small portion of a rod. G
The angular momentum LO of the whole rod and its components
along the axis of rotation and parallel to the x-y plane obtained by
integrating both the members of the above equation are shown in the above
figure. Thus we have
G G G
LO = ω ∫ r 2 dm − ω ∫ zrdm

Substituting I z = ∫ r 2dm for moment of inertia of the body about the z-axis in the
above equation, we have
G G G
LO = I z ω − ω ∫ zrdm [8.42]
G
The first term of the above equation is along the z-axis and represents Lz which
is the z-component of the angular momentum of the body about the origin.
G G
Lz = I z ω [8.43]
41

G
In the second term position vector r is measured from the center of the
circular path followed by every portion like P, thus the second term will always
G
be in the x-y plane and represent the component Lxy .
G G
Lxy = −ω ∫ zrdm [8.44]

It also easy to conceive that if the body is symmetrical about the axis of
rotation, for every portion of mass dm there will be a diametrically opposite
G
similar portion; therefore the integral ∫ zrdm carried over the whole body will
reduce to zero. Thus for bodies symmetric about the axis of rotation the total
angular momentum about the origin equals remains parallel to the axis of
rotation.
In the next section we will establish that net torque of all the external
forces equals to the time rate of change in angular momentum. If we consider
G
rate of change of angular momentum LO , the rate of change in its axial
G
component Lz will always be along the axis of rotation but the rate of change
G
in Lxy will always be perpendicular to the axis. In section 2.1, it has been
established that in plane motion the net torque is always along the axis of
rotation, therefore in plane motion the net angular momentum must also be
along the axis of rotation. Expressed in eq. [8.43], it is represented by the axial
G
component Lz and is known as angular momentum about the axis of rotation.

Like torque the angular momentum is also defined about a point, but for
analyzing plane motion i.e. rotation about fixed axis or axis in translation we
consider torque as well angular momentum about the axis.

Angular Momentum of a In this section we discus how to express angular momentum of body in plane
Rigid Body in plane motion. Till so far we have learnt that equations of kinematics, torque
Motion. equation and kinetic energy of a body in plane motion can be expressed by
representing the plane motion as superposition of translation of its mass
center and simultaneous centroidal rotation. We will seek for whether or not
can we represent the angular momentum too as superposition of translation of
its mass center and simultaneous centroidal rotation? We also develop
expressions for writing angular momentum in fixed axis rotation and rotation
about an instantaneous axis.
G
G vC
y vi / C

G
mi vi Superposition of translation of mass center and simultaneous Centroidal
G
ri / C
G Rotation. Consider a rigid body of mass M in plane motion. Its motion can be
vC
G analyzed as superposition of translation of its mass center and simultaneous
ri C
centroidal rotation. Characteristics of plane motion can satisfactorily be
G
rC represented by a laminar section of the body perpendicular to the axis of
rotation as shown in Fig. 8.25. Let one of its particles of mass mi is moving
O x G G
Fig. 8.25 with velocity vi , mass center C with velocity vC and the body is rotating with
angular velocity ω in clockwise sense relative to an inertial frame represented
G G
by a coordinate system in the figure. Vectors ri and rC represent position
G
vectors of the particle mi and the mass center and the vector ri / C represents
position vector of mi relative to the mass centre. An axis of rotation if passes
through the origin, must be perpendicular to the plane of motion of the
particles of the body and coincide with the z-axis. We denote angular
G
momentum of the body about this axis by vector Lo . The angular momentum
of the whole body about an axis is sum of angular momentums of all of its
particles about the same axis, so we have
G n
G G
Lo = ∑ ri × ( mivi )
i =1
42

G G G
Substituting relative velocity equation vi = vC + vi / C in the above one and
G G
thereafter ∑ miri = MrC in the first term of the obtained expression, we have
G G G G G
Lo = MrC × vC + ∑ ri × ( mivi / C )
G G G G
Substituting ri = rC + ri / C and then ∑mv i i/C = 0 , we have
G G G G G
Lo = MrC × vC + ∑ ri / C × ( mivi / C )
G G G
Substituting vi / C = ω × ri / C in the above equation and simplifying vector
G G G G G G G G
triple product term ri / C × (ω × ri / C ) , by using property a × (b × c ) = ( a ⋅ c ) b − ( a ⋅ b ) c ,
G G G G

we can write
G G G G G G G
( )
Lo = MrC × vC + ω ∑ ri2/C mi − ∑ ( ri / C ⋅ ω )ri / C mi
G G
Since ri / C ⋅ ω = 0 , the last term vanishes. Rearranging the first term and
substituting I C = ∑ ( ri2/Cmi ) for moment of inertial about the axis through O, we
have
G G G G
Lo = rC × ( MvC ) + I C ω [8.45]

The first term of the above equation represent angular momentum due to
translation of the mass center and the second term represents angular
momentum in centroidal rotation. The equation reveals that if a body is in
plane motion its angular momentum about the axis passing through the origin
equals to the vector sum of angular momentum due to translation of the mass
center and angular momentum due to centroidal rotation.

Rotation about the instantaneous axis of rotation. Consider a body in plane


G G
vi motion represented in Fig. 8.26, in which velocity vectors vC of a particle mi
mi G
G and vi of the mass center C relative to the coordinate system are shown. If the
ri / C G
vC point P is the ICR, the instantaneous axis of rotation passes through it and is
G G
G
ri / P
C
G perpendicular to the x-y plane. The position vectors ri / P and rC / P must the
rC / P
perpendicular to the corresponding velocity vectors. The velocity vectors of the
P G G G G
ω particle mi and mass center bear the relations vi = ω × ri / P and
G G G
vC = ω × rC / P respectively. Since ICR is at instantaneous rest velocity of any
Fig. 8.26 particle relative to ICR must be same as relative to the inertial frame.
G G
Therefore we can safely assume origin and the ICR. It will transform ri to ri / P ,
G G
rC to rC / P and leave all the velocity terms the same. Making these
G G G
transformations and substituting vC = ω × rC / P in eq. [8.45], we have
G G G G G
LP = rC / P × ( M ω × rC / P ) + I C ω
G G G
Simplifying vector triple product term rP / C × (ω × rP / C ) , by using property
G G G G G G G G G
( ) ( )
a × b × c = ( a ⋅ c ) b − a ⋅ b c , we can write
G G G G
(
LP = MrC2/ P ω + IC ω = MrC2/ P + I C ω )
Making use of the theorem of parallel axes I P = MrC2/ P + IC , we have
G G
LP = I P ω [8.46]

The above equation reveals that the angular momentum of a rigid body in
plane motion can also be expressed in a single term due to rotation about the
instantaneous axis of rotation.
43

Rotation about the instantaneous axis of rotation. The instantaneous axis of


rotation is at instantaneous rest. If we assume the body described in Fig. 8.26
rotating about a fixed axis through point P, eq. [8.46] can be used to express its
kinetic energy

4.3.Rotational Equivalent Differentiating terms on both the sides of eq. [8.45] with respect to time, and
G G G G G G
of the Newton’s Laws of making substitution of vC = drC dt , aC = dvC dt and α = dω dt we have
Motion. G
dLo G G G G G
= vC × ( MvC ) + rC × MaC + I Cα
dt
The first on the right hand side vanishes, so we can write
G
dLo G G G
= rC × MaC + I Cα [8.47]
dt
From eq. [8.28] and [8.47], we have
G
G dLo
∑τ o = dt [8.48]

The above equation though developed for plane motion only yet is valid for
rotation about an axis in rotation also. It states that the net torque about the
origin of an inertial frame equals to the time rate of change in angular
momentum about the origin and can be treated as a parallel to Newton’s
second law which states that net external force on a body equals to time rate of
change in its linear momentum.

4.4.Angular Impulse Rearranging the terms and integrating both the sides obtained form the eq.
Momentum Principle. [8.48], we can write
t2 G G G
∑ ∫ τ odt = Lo2 − Lo1
t1
[8.49]

Making use of eq. [8.40], we have


G G G
∑ J o,1→2 = Lo2 − Lo1 [8.50]

The term on the left hand side is the net angular impulse about the origin
in the time interval t1 to t2, the first term of the right hand side is the angular
momentum about the origin at the instant t2, and the second term on the right
hand side is the angular momentum about the origin at the instant t1. It is
known as angular impulse momentum principle and states that increment in
the angular momentum of a body about a point in a time interval equals to the
net angular impulse of all the external forces acting on it during the concerned
time interval.
For the ease of application the eq. [8.50] is rearranged as
G G G
Lo1 + ∑ J o ,1→2 = Lo 2 [8.51]

The above equation states the principle of angular momentum and impulse
in another way as net angular impulse of all the external forces about a point
during a time interval when added to angular momentum of a body about the
same point at the beginning of an interval of time we get angular momentum of
the body about the same point at the end of the interval concerned.
Like linear impulse momentum principle, the angular impulse momentum
principle provides us solution of problems concerned with change in angular
velocity in a time interval or change in angular velocity during very short
interval interactions.
44

Plane motion is combination of translation and simultaneous centroidal


4.5.Impulse Momentum rotation. Linear momentum and angular momentum serve as measures of
Principle for Plane motion
amount of translation and rotation motion respectively. The external forces
of a Rigid Body.
acting on a rigid body can change its state of translation as well as rotation
motion which is reflected by change in linear as well as angular momentum
according to the principles of linear impulse and momentum and angular
impulse and momentum.

G
Consider a rigid body of mass M in plane
∫ F dt
1
G
motion. Its moment of inertial about the
G ∫ F dt
i centroidal axis perpendicular to plane of
G MvC1 G G G
ICω1 IC ω2 G motion is IC. Let vC1 and ω1 represent velocity
MvC 2
C C of its mass center and its angular velocity at
G
the beginning of a time interval t1 to t2. Under
∫ F2dt G G G G
∫ Fndt the action of several forces F1 , F2 ……. Fi
G
Linear and angular Impulse of all the Linear and angular ….. Fn during the time interval its mass
momentums at the forces during time momentums at the
instant t1. interval t1 to t2. instant t2. center velocity and angular velocity become
G G
vC 2 and ω2 respectively. The adjacent figure
Fig. 8.27 Impulse Momentum Diagram.
shows strategy representing how to write
equations for linear and angular impulse
momentum principles.
The eq. [8.51] represents angular impulse momentum principle about the
origin. While applying the principle it becomes simpler to consider translation
of the mass center and centroidal rotation separately. Thus in an alternative
way we apply linear impulse momentum principle for translation of the mass
center and angular impulse momentum principle for centroidal rotation.

Translation of mass center: Linear impulse momentum principle.


G G G
p1 + ∑ I mp1→2 = p2 [8.52]
G G G G
Here p1 = MvC1 and p2 = MvC 2 represent linear momentums at the
G
beginning and end of the time interval and ∑I mp1 → 2 stands for
impulse of all the external forces during the time interval.
Centroidal rotation: Angular impulse momentum principle.
G G G
LC1 + ∑ J C ,1→2 = LC 2 [8.53]
G G G G
Here LC1 = I Cω1 and LC 2 = ICω2 represent angular momentums about
the centroidal axis at the beginning and end of the time interval
G
and ∑ J C ,1→2 stands for angular impulse of all the external forces
about the centroidal axis during the time interval.
Brake pad Example 8.35. A disk of mass M and radius R is rotating with angular velocity ωo
about its centroidal axis of symmetry. There is a braking mechanism installed to stop
the disk. When the brake is applied, the brake pad presses the rim of the disk with a
normal force N. If the coefficient of kinetic friction between the brake pad and the rim
of the disk is μ, find the time in which the disk will stop rotating.
Solution. The forces acting on the disk are the normal reaction form the brake pad,
Fig. Ex. 8.35 force of kinetic friction between the brake pad, weight of the disk and reaction from the
axel. Its weight, normal reaction and reaction force from the axel do not apply any
torque about the axel and play no role in stopping the disk. It is the angular impulse of
the force of kinetic friction which stops the disk.
μ Nt
The impulse momentum diagram is shown in the adjacent
figure. Applying angular impulse momentum principle
ICωo 0 according to eq. [8.53], we have
G G G
LC1 + ∑ J C ,1→2 = LC 2 → I Cωo − μ Nt = 0
45

Substituting I C = 12 MR 2 , we have

MR 2ωo
t= Ans.
μN

Example 8.36. A uniform disk of mass M and radius R rotating with angular velocity
ωo about a vertical axis passing through its center and perpendicular to its plane is
placed gently on a rough horizontal ground, were coefficient of friction is μ. How long it
will take to stop.
Solution. Refer the worked out example 8.12. The torque of friction forces is
Fig. Ex. 8.36
τ C = 32 μ MgR (1)

The angular impulse of the torque of friction is responsible to stop the disk. Applying
angular impulse momentum principle, we have
G G G
LC1 + ∑ J C ,1→2 = LC 2 → I C ωo − τ C t = 0

Substituting I C = 12 MR 2 and τ C from eq. (1), we have

3Rωo
t= Ans.
4μ g

Example 8.37. A uniform sphere of mass m and radius r is projected along a rough
horizontal floor with linear velocity vo and no angular velocity. The coefficients of
vo kinetic and static frictions are represented by μs and μk respectively.
(a) How long the sphere will slide on the floor before it starts rolling.
(b) How far the sphere will slide on the floor before it starts rolling.
(c) Find the linear and angular velocities of the sphere when it starts rolling.
(d) Find the work done by frictional forces during the process and thereafter.
Fig. Ex. 8.37
mg Solution. When the sphere touches the floor it is on translation motion. All the
points including the bottom one are moving with the same velocity vo. Thus the bottom
y
point which makes the contact with the floor slide on it causing kinetic friction to act in
x backward direction. In the adjacent figure the forces acting on the sphere are shown.
C
Here mg represent weight, N the normal reaction from the ground and fk .

Since the sphere has no vertical component of acceleration, by applying Newton’s law
fk we have
N
∑F
y = 0→ N = mg

The kinetic friction fk = μ N = μmg (1)

The only force which applied torque about torque about the centroidal axis is the
kinetic friction. Angular impulse of torque of kinetic friction increases the angular
velocity ω and impulse of kinetic friction decreases the mass center velocity vC till both
bear following condition required fro rolling. Thereafter the sphere will continue to roll
with the uniformly.
vC = ω r (2)

y In the adjacent impulse momentum diagram only the


p1 = mvo p2 = mvC impulse of kinetic friction is shown
x
C C C Translation of mass center: Applying linear impulse
LC1 = 0 LC 2 = IC ω momentum principle in x direction, we have
G G G
p1 + ∑ I mp1→2 =p2 → p1 − fkt = p2
∫ f dt = f t
k k

Substituting p1, p2 and fk from eq. (1), we have

vC = vo − μ gt (3)

Centroidal rotation: Angular impulse momentum principle about the centroidal axis.
46

G G G
LC1 + ∑ J C ,1→2 = LC 2 → 0 + fk rt = I C ω

Substituting 2
5
mr 2 for IC and fk from eq. (1), we have

5μ gt
ω= (4)
2r

(a) Substituting values of vC and ω form eq. (3) and (4) into eq. (2), we have

2vo
Time when rolling starts t= (5) Ans.
7μ g

(b) Eq. (3) reveals that the mass center is in uniformly retarded motion. So its
displacement in time t, when it starts rolling is given by the following equation.
x= 1
2 (vo + vC ) t → Substituting values for vC and t from eq. (3) and (5)
respectively we have

12vo2
x= (6) Ans.
49 μ g

(c) Linear and angular velocities of the sphere when it starts rolling can be obtained
by substituting t from eq. (5) into (3) and (4) respectively.
Linear velocity when rolling starts vC = 75 vo (7) Ans.

5vo
Angular velocity when rolling starts ω= (8) Ans.
7r

(d) Work done by a force depends on displacement of point of application or


displacement of the particle on which force is applied. The particle of the body in
contact with the ground on which force of kinetic friction acts continuously changes;
therefore it is recommended to calculate work done with the help of work energy
theorem instead of using eq. 5.2.
Kinetic energy in the initial position at the instant t1,

K = 12 mvC2 + 12 I C ω 2 → K1 = 12 mvo2 (9)

Since in the beginning angular velocity is zero.


Kinetic energy in the final position at the instant t2,

K = 12 mvC2 + 12 I C ω 2 → Substituting values of vC and ω form eq. (7) and (8) and
2
5
mr 2 for IC, we can write

35mvo2
K2 = (9)
98

Only force of kinetic friction does work during sliding. Denoting it by Wf 1→2 in the
equation of work energy theorem, we have
K1 + W1→2 = K 2 → Wf 1→2 = K 2 − K1

Substituting values of K1 and K2 form eq. (8) and (9), we


have

Wf 1→2 = − 17 mvo2 Ans.

4.6. Conservation of In this section we discus the principle applied to rigid body and can be
Angular Momentum. considered a special application of the more general principle − principle of
conservation of angular momentum of s system of particles. Here too we start
47

with the angular impulse momentum principle described by eq. [8.49], which is
reproduced again.
t2 G G G
∑ ∫ τ o dt = Lo2 − Lo1
t1
[8.54]

t2 G G G
If ∑∫ t1
τ o dt = 0 ⇒ Lo1 = Lo 2 [8.55]

The above statement is known as principle of conservation of angular


momentum and states that if net angular impulse of all the external forces
about an axis vanishes in a time interval, the angular momentum about that
axis remains constant in that time interval.
The condition of zero net angular impulse required for conservation of
angular momentum can be fulfilled in the following cases.
ƒ If no external force acts, the angular impulse about all axes will be zero
and hence angular momentum remains conserved about all axes.
ƒ If net torque of all the external forces or torques of each individual force
about an axis vanishes the angular momentum about that axes will be
conserved.
ƒ If all the external forces are finite in magnitude and the concerned time
interval is infinitely small, the angular momentum remain conserved.
ƒ If a system of rigid bodies changes its moment of inertia by changing its
configuration due to internal forces only its angular momentum about any
axes remains conserved. If we denote the moment of inertias in two
configurations by I1 and I2 and angular velocities by ω1 and ω2, we can write
G G
I1ω1 = I 2ω2 [8.56]

The principle of conservation of angular momentum governs a wide range


of physical processes from subatomic to celestial world. The following examples
explicate some of these applications.

Spinning Ice Skater. A spinning ice skater and ballet dancers can control
her moment of inertia by spreading or bringing closer her hands and make use
of conservation of angular momentum to perform their spins. In doing so no
external forces is needed and if we ignore effects of friction from the ground
and the air, the angular momentum can be assumed conserved. When she
spreads her hand or leg away, her moment of inertia decreases therefore her
angular velocity decreases and when she brings her hands or leg closer her
moment of inertia increases therefore her angular velocity increases.
Fig. 8.28 Spinning ice skater.

Student on rotating turntable. The student, the turntable and


dumbbells make an isolated system on which no external torque
acts, if we ignore friction in the bearing of the turntable and air
friction. Initially the student keeps his arm stretched and the
turntable is rotating as shown in Fig. 8.29. When the student pulls
his arms and brings the dumbbells close to his body the moment of
inertia of the system decreases. In this action no external force
Larger moment of Smaller moment of
inertia and smaller inertia and larger
hence torque is applied, therefore angular momentum remains
angular velocity. angular velocity. conserved and by using eq. [8.56], we can say that the angular
Fig. 8.29 velocity of the student increases.
48

The springboard Diver. A springboard diver after leaving the board performs
acrobatic activities in the air. The diver leaves the board in such a manner
that the angular impulse of the normal reaction N from the board about her
G
mass center imparts her angular momentum LC pointing into the plane of the
paper as shown by encircled cross. While she is in air the only force is her

C C G
G L
LC

mg
mg

N
N

Fig. 8.29 (a) The diver leaves the board, the normal reaction Fig. 8.29 (b) After leaving the board she folds her body till
G
from the board imparts her angular momentum LC about her position shown by third snapshot and thereafter she spreads
her body. In doing so she first reduces her moment of inertia so
mass center. Her mass center follows parabolic trajectory and
her angular velocity till the position shown in the third
she rotates about her mass center at constant angular velocity.
snapshot, thereafter she increases her moment of inertia so
decreases her angular velocity because her angular momentum
remains conserved..

weight mg, whose torque about the mass center is zero, therefore her angular
momentum remains constant and mass center C follows a parabolic trajectory
shown by dotted cure. If the diver keeps her body straight during the flight,
her moment of inertia about the centroidal axis remains constant so her
angular velocity as shown in Fig. 8.30 (a). In Fig (b) it is shown that the diver
first folds her body till the third snapshot is taken, her moment of inertia
decreases, and therefore according to eq. [8.56] her angular velocity increases.
The increasing angular velocity enables her to complete about 1.5 revolutions,
while in the previous trial with straight body she had competed close to half
revolution till this moment. After the instant shown in the third snapshot she
straightens her body and decreases her angular velocity by increasing her
moment of inertia.

Example 8.38. Consider the disk A of moment of inertia I1 rotating freely in horizontal
plane about its axis of symmetry with angular velocity ωo. Another disk B of moment of
B
inertia I2 held at rest above the disk A. The axis of symmetry of the disk B coincides
ωο
with that of the disk A as shown in the figure. The disk B is released to land on the disk
A
A. When sliding stops, what will be the angular velocity of both the disks?
Solution. Both the disks are symmetric about the axis of rotation therefore doe not
Fig. Ex. 8.38 require any external torque to keep the axis stationary. When the disk B lands on A
slipping starts. The force of friction provides an internal torque to system of both the
disk. It slows down rotation rate of A and increases that of B till both acquire same
angular velocity ω.
ω
ω3ο
A

vi = vo − ωr

Fig. Ex. 8.39


49

Since there is no external torques on the system of both the disks about the axis of
rotation, the total angular momentum of the system remains conserved. The total
angular momentum of the system is the sum of angular momentum of both disks.
Denoting the angular momentum of the disk A before B lands on it and long after
G G G G
slipping between them stops by symbols LA1 , LB1 , LA 2 and LB 2 respectively, we can
express conservation of angular momentum by the following equation.
G G G G I1ωo
LA1 + LB1 = LA 2 + LB 2 → I1ωo + 0 = I1ω + I 2ω ⇒ ω = Ans.
I1 + I 2

Example 8.39. A disk of moment of inertia I is rotating freely with angular velocity ωo
about its vertical axis of symmetry as shown in the figure. An insect of mass m lands
with negligible horizontal velocity on the disk at a distance r from its center.
(a) Find the angular velocity of the disk immediately after the insect lands.
(b) If the insect starts crawling with a speed vo relative to the disk on a circular path of
radius r in clockwise direction as viewed from the top, what will be the angular
velocity of the disk?
Solution. As the insect lands asymmetry produced in mass distribution in the disk-
insect system requires an additional torque required to keep the axis of rotation
vertical. This torque is perpendicular to the axis and has no component about it. Thus
there is no external torque about the axis on the insect-disk system. The angular
momentum of the system about the axis must remain conserved.
G G
(a) Denoting the angular momentum of the disk and the insect by Ld and Li with
additional subscripts 1 to represent situation before the insect lands and 2 to
represent situation immediately after the insect lands, we can express conservation
of angular momentum of the system by the following equation.
G G G G
Ld1 + Li1 = Ld 2 + Li 2 (1)

The insect has no horizontal velocity before it lands and, therefore its angular
momentum about the axis before it lands is zero. Immediately after it lands it
sticks to the disk and moves together with the disk with a new angular velocity say
ω2. The moment of inertia of the insect equals to mr 2 .

Li1 = 0 and Li 2 = mr 2ω2

I ωo
Substituting above information in eq. (1), we have ω2 = (2) Ans.
I + mr 2
(b) The angular momentum of the insect-disk system about the axis remains conserved
whether the insect rests or crawls on the disk. Adding subscript 3 to denote angular
momentum of the disk and the insect after the insect starts crawling, we can
express the conservation of angular momentum by the following equation.
G G G G G G
Ld1 + Li1 = Ld 2 + Li 2 = Ld 3 + Li 3 (3)

As shown in the adjacent figure, the insect crawls with velocity vo in the clockwise
sense relative to the disk which is rotation in anticlockwise sense. Let the velocity
of the point of the disk underneath the insect is less than the velocity of the insect
relative to the disk. The velocity of the insect relative to the ground at the instant
shown is
vi = vo − ω3r (4)

The angular momentum of the insect about the axis is


Li 3 = mrvi (5)
O r C
I ωo + mrvo
ωo From eq. (2), (3), (4) and (5), we have ω3 = Ans.
I + mr 2

Example 8.40. A boy standing on a stationary turntable holds a disk of mass m in his
hands with its centroidal axis O in vertical position as shown in the figure. The disk is

Fig. Ex. 8.40


50

spinning at angular velocity ωo about its centroidal axis C. Moment of inertia of the
disk about its centroidal axis is Io and the moment of inertia of the boy and turntable
about axis O is I. The boy inverts the disk. Find the angular velocity of the boy.
Solution. To invert the disk no external force is applied, therefore angular
momentum of the system about any axis remain conserved.
We denote angular momentum of the boy and turntable about the axis O before and
after inverting the disk by LB1 and LB2 and that of the disk by LD1 and LD2. The angular
velocity of the boy and turntable after inverting the disk is ω in the clockwise sense. Let
the clockwise direction as viewed form top is positive.
After inverting the disk the mass center of the disk will move on
LB 2 a circular path of radius r with velocity
LD1 ω vC = rω
O r C O r C
ωo ωo Angular momentums about the axis O before inverting the disk

Boy and the turntable LB1 = 0


ωo
LD 2
Disk LD1 = I o ωo

Angular momentums about the axis O after inverting the disk

Boy and the turntable LB 2 = I ω


Before inverting the disk After inverting the disk
Disk LD 2 = mr 2 ω − I o ωo

Applying conservation of angular momentum of the whole system consisting of the boy,
the turntable and the disk about the axis O, we have
LB1 + LD1 = LB 2 + LD 2 → Substituting values form the above equations, we have

Angular velocity of the boy and the turntable after the disk is
inverted
2 I o ωo
ω= Ans.
I + mr 2

Example 8.41. A cube of mass m and edge length A can slide freely on a smooth
vo horizontal floor. Moving on the floor with velocity vo, it strikes a long obstruction PP of
P small height. The obstruction is parallel to the leading bottom edge of the cube. The
leading bottom edge gets pivoted with the obstruction and the cube starts rotating.
P Determine angular velocity of the cube immediately after the impact.
Solution. Before the impact the, there is no external force in the horizontal direction
and the cube slides with uniform velocity vo, and during the impact reaction forces of
the obstruction stops its leading bottom edge and cause it to rotate about the its leading
bottom.
During the impact external forces acting on the cube are its weight, the normal reaction
from the ground and reaction from the obstruction. The weight and the normal reaction
from the ground both are finite in magnitude and the impact ends in infinitesimally
small time interval so their impulses and angular impulses about any axes are
negligible. It is the reaction from the obstruction which has finite impulse during the
impact. Its horizontal component changes the momentum of the cube during the
impact, but its angular impulse about the obstruction is zero, therefore the angular
momentum of the cube about an axis coincident with the leading bottom edge remain
conserved.
Let the velocity of the mass center and angular velocity of the cube immediately after
the impact are vCo and ωo. These velocities are shown in the adjacent figure.

We denote the angular momentum of the cube about and axis coincident with the
obstruction edge before and after the impact by LP1 and LP2.
vo

vo
C C
P ω P

Immediately before the Immediately after the


impact. impact.
51

Applying principle of conservation of angular momentum about an axis coincident with


the obstruction, we have
G G G G G
LP1 = LP 2 → rC / P × mvo = I P ω

Using theorem of parallel axes for moment of inertia IP about the leading bottom edge,
we get I P = I C + mrC2 / P = 32 mA2 . Substituting this in the above equation, we have

Angular velocity immediately after the impact


3vo
ωo = (1) Ans.
4A

Example 8.42. Consider two identical disks A and B with their axels held horizontal.
ωo
Both of them can rotate about their axels freely. Initially the disk A is rotating with
angular velocity ωo in clockwise direction. Both of them are brought closer gently to
A B
touch each other.
(a) Is total angular momentum of the system conserved?
(b) When slipping stops what will be the angular velocity of each of the disks?
Fig. Ex. 8.41
(c) Solve the previous part assuming radius of the disk B double of that of disk A but
same mass.
Solution. When the disks touch each other slipping starts and force of kinetic friction
f acts upwards on the periphery of disk A and downwards on the periphery of the disk B
slowing down the disk A and speeding up B. When slipping stops the points on both
the disks in contact move with the same velocity.
f
(a) When the disks touch each other frictional forces create reaction forces R on the on
R the axels of each disk as shown in the adjacent figure. The figure shows that we
A P B cannot find any axes about which net torque of all these forces will be zero;
R therefore angular momentum of the two disk system is not conserved. Ans.
f

(b) When slipping stops the points on both the disks in contact move with the same
ω ω velocity and both the disks are of the equal radii; therefore both of them will rotate
A B with the same angular speed ω as shown in the adjacent figure.
The problem involves change in angular velocity in a time interval, thus principle
of angular impulse and momentum has to be applied on each of the disk separately.
The reaction of the axel and weight do not contribute any torque so their impulses
are not shown in the impulse momentum diagram.
Applying principle of angular impulse and momentum to
fδt disk A according the impulse momentum diagram shown, we
Ιωo Ιω
have
A A A
G G G
LC1 + ∑ J C ,1→2 = LC 2 → I ωo − frδ t = I ω (1)

Here r is the radius of the disk.


Applying principle of angular impulse and momentum to
0 Ιω disk A according the impulse momentum diagram shown, we
B B B have
G G G
fδt LC1 + ∑ J C ,1→2 = LC 2 → frδ t = I ω (2)

From eq. (1) and (2), we have


Final angular speed of both the disk ω = 12 ωo Ans.

(c) If radius of the disk B is double of that of disk A, its moment of inertia will four
times of that of A and final angular velocity will be half of that of A. Substituting
these information in eq. (1) and (2), we have
ωo ωo
Final angular speeds of the disks ωA = and, ωB = Ans.
2 4
52

5. Eccentric Impact. When two smooth bodies undergo central impact the line of impact, which is
the common normal at the point of contact, passes through the mass centers of
both the bodies and none of them can change state of rotation motion of other.
Consider impact of two smooth bodies A and B such that the mass center CB of
CB Line of B does not lie on the line of impact as shown in Fig. 8.28. Since bodies are
Impact
smooth their mutual forces must act along the line of impact. The reaction
A B force of A on B does not passes through the mass center of B as a result state
CA
of rotation motion of B changes during the impact. In eccentric impacts mass
Fig. 8.28 centers of either or both the bodies undergoing impact do not lie on the line of
impact. It involves change in state of rotation motion of either or both the
bodies.

Coefficient of restitution Consider eccentric impact of two smooth bodies of masses m1 and m2. There are
in eccentric impact. no external forces acting on the bodies. Before the impact they are rotating
with angular velocities ω1 and ω2 and their mass centers A and B are moving
G G
with velocities v1 and v2 relative to an inertial frame in free space. When they
G
G v2 collide, particle P of the first body and particle Q of the second body come in
v1 ω2
contact. The line NN is the common normal drawn at the point of contact as
ω1 B
shown in Fig. 8.29. When they come into contact they press each other with a
A
Q N force D during the period of deformation and with a force R during the period
N P of restitution. Both these forces act along the common normal. Let at the end
of period of deformation angular velocities become ω1o and ω2o and velocities of
G G
Fig. 8.29 mass centers become u1 and u2 respectively due to action of impulse
∫ Ddt during the period of deformation. During the period of restitution
impulse ∫ Rdt changes angular velocities to ω' 1
and ω'2 and mass center
G G
velocities to v1' and v2' .

The impulse momentum diagrams of the first body during the period of
deformation and period of restitution are shown in Fig. 8.30. We represent
velocity component along the common normal by adding a subscript n to the
corresponding quantity.
G
m1v1
G
m1u1 Applying linear impulse momentum principle along the
common normal during the period of deformation and
I1ω1 I1ω1o
A
A
A
restitution separately, we have
r ∫ Ddt G
G G
N P N P N P p1 + ∑ I mp1→2 = p2 → mv1n − ∫ Ddt = m1u1 [8.57]

Fig. 8.30 (a) Period of Deformation. mu1n − ∫ Rdt = m1v1' [8.58]


G
m1u1
G
m1v1' Applying angular impulse momentum principle for
I1ω1o
centroidal rotation during periods of deformation and
A I1ω'

A
1
A restitution, we have
r ∫ Rdt G G G
N P N P N P LC1 + ∑ J C ,1→2 = LC 2 → I1ω1 − r ∫ Ddt = I1ω1o [8.59]

Fig. 8.30 (b) Period of Restitution. I1ω1o − r ∫ Ddt = I1ω1' [8.60]

We recall that the coefficient of restitution e is defined


as the ratio

e=
∫ Rdt [8.61]
∫ Ddt
Substituting for the two impulses from eq [8.57] and [8.58] into eq. [8.61], we
have
53

u1n − v1' n
e= [8.62]
v1n − u1n

Similarly from eq [8.59], [8.60] and eq. [8.61], we have


ω1o − ω1'
e= [8.63]
ω1n − ω1o
Multiplying by r the numerator and denominator of eq. [8.63] and adding to
numerator and denominator of eq.[8.62], we have

(u1n + rω1o ) − (v1' n + rω1' )


e=− [8.64]
(v1n + rω1 ) − ( u1n + rω1o )
Applying relative motion equation, we have
vpn = v1n + rω1 ≡ Component of velocity of point P along the
commom normal before the impact.

u pn = u1n + rω1o ≡ Component of velocity of point P along the commom


normal at the end of period of deforamtion.

v'pn = v1' n + rω1' ≡ Component of velocity of point P along the


commom normal after the impact.

Substituting above values in eq. [8.64], we have


u pn − v'pn
e= [8.65]
vpn − u pn

In similar fashion we can obtain relation for coefficient of restitution in


terms of normal component of velocities of point Q by applying impulse
momentum principle to second body.
'
vQn − uQn
e= [8.66]
uQn − vQn

At the end of period of deformation the points P and Q move with the same
normal component of velocities, therefore we have u pn = uQn . Now adding
numerators and denominators of eq. [8.65] and [8.66], we have
' '
vQn − vPn
e= [8.67]
vpn − vQn

Care must be taken that the above equation expresses coefficient of


restitution in terms of normal component velocities of the points on the two
bodies which make contact during the impact and not in terms of normal
component of velocities of mass centers of the bodies.
The eq. [8.67] remains valid when one or both of the colliding bodies are
hinged.
Problems of Eccentric Problems of eccentric impact can be divided into two categories. In one
Impact. category both the bodies under going eccentric impact are free to move. No
external force act on either of them. There mutual forces are responsible for
change in their momentum and angular momentum. In another category
either or both of the bodies are hinged.

Eccentric Impact of bodies free to move. Since no external force acts on the
two body system, we can use principle of conservation of linear momentum,
principle of conservation of angular momentum about any point and concept of
coefficient of restitution as described in eq. [8.67]. However care must be taken
in selecting the point about which we write equation of conservation of angular
54

momentum. The eq. [8.45] describes angular momentum of a rigid body about
the origin. It reveals that the point about which we write angular momentum
must be at rest relative to the selected reference frame and as far as possible
its location should be selected in order to make zero the first term involving
moment of momentum of mass center.

Eccentric Impact of hinged bodies. When either or both of the bodies are
hinged the reaction of the hinge during the impact act as external force on the
two body system, therefore linear momentum no longer remain conserved and
w cannot apply principle of conservation of linear momentum. When both the
bodies are hinged we cannot also apply conservation of angular momentum,
and we have to use impulse momentum principle on both the bodies separately
in addition to eq. [8.67]. But when one of the bodies is hinged and other one is
free to move, we can apply conservation of angular momentum about the hinge
and eq. [8.67].

Example 8.43. A uniform rod of mass m and length A is suspended from a fixed
O
support and can rotate freely in the vertical plane. A small ball of mass m moving
horizontally with velocity vo strikes elastically the lower end of the rod as shown in the
figure. Find the angular velocity of the rod and velocity of the ball immediately after
the impact.
vo Solution. The rod is hinged and the ball is free to move. External forces acting on the
Fig. Ex. 8.42
rod ball system are their weights and reaction from the hinge. Weight of the ball as
well as the rod are finite and contribute negligible impulse during the impact, but
O ω' O impulse of reaction of the hinge during impact is considerable and cannot be neglected.
Obviously linear momentum of the system is not conserved. The angular impulse of the
reaction of hinge about the hinge is zero. Therefore angular momentum of the system
about the hinge is conserved. Let velocity of the ball after the impact becomes vB' and
angular velocity of the rod becomes ω’.
vo v'
We denote angular momentum of the ball and the rod about the hinge before the
Before the impact Immediately after impact by LB1 and LR1 and after the impact by LB2 and LR2.
the impact
Applying conservation of angular momentum about the hinge, we have
G G G G
LB1 + LR1 = LB 2 + LR 2 → mvoA + 0 = mvB' A + I oω '

Substituting 1
3
M A2 for Io, we have

3mvB' + M Aω ' = 3mvo (1)

The velocity of the lower end of the rod before the impact was zero and immediately
after the impact it becomes Aω ' towards right. Employing these facts we can express
the coefficient of restitution according to eq. [8.67]
' '
vQn − vPn
e= → Aω ' − vB' = evo (2)
v pn − vQn

From eq. (1) and (2), we have

Velocity of the ball immediately after the impact vB' =


(3m − eM ) vo Ans.
3m + M

3 (1 + e ) mvo
Angular velocity of the rod immediately after the impact ω ' = Ans.
(3m + M ) A

vo
A Example 8.44. A uniform rod AB of mass M and length A is kept at rest on a smooth
horizontal plane. A particle P of mass mo moving perpendicular to the rod with velocity
vo strikes the rod one of its ends as shown in the figure. If the particle stops moving
vP' A

Fig. Ex. 8.43 vC'


B
ω'
C

B
55

after the impact, derive suitable expressions for the coefficient of restitution, velocity of
mass center of the rod and angular velocity of the rod immediately after the impact.
Solution. Both the bodies can move freely in the horizontal plane, therefore no
horizontal external fore acts on the particle-rod system. The linear momentum as well
as angular momentum about any axis normal to the plane is conserved.
Let the velocity of the particle, angular velocity of the rod and velocity of the mass
center of the rod immediately after the impact are vP' towards right, ω' in clockwise
sense and vC' towards right as shown in the adjacent figure. Using relative motion
equation, we can express the velocity of the end A of the rod.

v'A = vC' + 12 ω 'A (1)


G G
We denote linear momentum of the particle and rod before the impact by pP 1 and pC1
G G
and immediately after the impact by pP 2 and pC 2 respectively.

Applying conservation of linear momentum, we have


G G G G
pP 2 + pC 2 = pP1 + pC1 → mvP' + MvC' = mvo (2)

The above equation shows that the mass center of the rod will move toward the right. If
we write angular momentum of the rod about a stationary point O, which is in line with
the velocity vC' , the first term involving moment of momentum of rod vanishes and only
angular momentum due to its centroidal rotation remains in the expression.
We denote angular momentum of the particle and the rod about the point O before the
impact by LP1 and LR1 and after the impact by LP2 and LR2.

A vP' A Applying conservation of angular momentum about the hinge, we have


vo
G G G G
LP 2 + LR 2 = LP1 + LR1 → 12 mvP' A + I Cω ' = 12 mvo A
ω' vC'
Substituting 1
12
M A2 for IC, we have
C

6mvP' + M Aω ' = 6mvo (3)


B B
Before the impact Immediately after The velocity of the end A of the rod before the impact was zero and immediately after
the impact the impact it becomes v'A towards right. Employing these facts we can express the
coefficient of restitution according to eq. [8.67]
' '
vQn − vPn
e= → v'A − vP' = evo
v pn − vQn

Substituting v'A form eq. (1), we have

2vC' + ω 'A − 2vP' = 2evo (4)

Eq. (2), (3) and (4) involves three unknowns vC' , ω ' and vP' , which can be obtained by
solving these equation.

⎛ 4m − eM ⎞
Velocity of the ball immediately after the impact vP' = ⎜ ⎟ vo Ans.
⎝ 4m + M ⎠

⎧⎪ m (1 + e ) ⎫⎪
Velocity of mass center of rod immediately after the impact vC' = ⎨ ⎬ vo Ans.
⎩⎪ 4m + M ⎭⎪

⎧⎪ 6m (1 + e ) ⎫⎪ vo
Angular velocity of the rod immediately after the impact ω' = ⎨ ⎬ Ans.
⎪⎩ 4m + M ⎪⎭ A

vo Example 8.45. A sphere of mass m and radius r rotating at angular velocity ωo strikes
the horizontal ground with velocity vo as shown in the figure. The coefficient of kinetic
C friction between the ground and the sphere is μ and the coefficient of restitution for the
ωo impact is e, Develop suitable expression for the velocity of mass center and angular
velocity of the ball immediately after the impact.

Fig. Ex. 8.44


56

Solution. The point of contact P and the mass center of the sphere have the same
vertical component of velocity before and immediately after the impact. The vertical
direction also coincides with the common normal at the point of contact. Let the normal
components of velocities of the point of contact P of the sphere before and immediately
'
after the impact are vo in the down downward direction and vCy in the upward
direction. The ground remains stationary in the selected laboratory frame. Assuming
sphere as the first body and the ground as the second body, the coefficient of restitution
can be expressed as
' ' '
vQn − vPn 0 − vCy '
e= → e= ⇒ vCy = evo (1)
v pn − vQn −vo − 0

The above equation shows that the normal component of velocity of the sphere becomes
reversed in direction and e times in magnitude similar to central impact. It happens
due to impulse of the normal reaction from the ground. Thus impulse of the normal
reaction has considerable effect therefore the impulse of the friction force will also have
considerable effects. The point of contact P slides towards the left, therefore force of
kinetic friction acts towards the right and impart the sphere a horizontal velocity
'
vCx and changes angular velocity to ω'.

y The impulse momentum diagram of the sphere during the


mvo '
mvCy impact is shown in the adjacent figure.
x
C C C Applying linear impulse momentum principle in y-direction,
ΙCω ICω '
' we have
mvCx
G G G
p1 + ∑ I mp1→2 =p2 → −mvo + ∫ Ndt = mvCy
'

∫ fdt = μ ∫ Ndt
'
Substituting vCy from eq. (1), we have
∫ Ndt
∫ Ndt = (1 + e ) mv
o (2)

Applying linear impulse momentum principle in x-direction, we have


G G G
p1 + ∑ I mp1→2 =p2 → 0 + μ ∫ Ndt = mvCx
'

Substituting ∫ Ndt form eq. (2), we have

'
vCx = μ (1 + e )vo (3)

From eq. (1) and (2), velocity of mass center of the ball immediately after the impact
G
vC' = μ (1 + e )voiˆ + evo jˆ Ans.

Applying angular impulse momentum principle about the centroidal axis.


G G G
LC1 + ∑ J C ,1→2 = LC 2 → 0 − r μ ∫ Ndt = I Cω '

Substituting ∫ Ndt form eq. (2), we have

5μ (1 + e )vo
ω ' = ωo − Ans.
2r

Potrebbero piacerti anche